You are on page 1of 62

COAGULATION & TRANSFUSION - 12

14. a 57-year-old man is undergoing an oesophagogastroduodenoscopy in theatre for an upper gastrointestinal


bleed. The nurse in the theatre goes to pick up blood from the hospital blood bank to be transfused
intraoperatively to the patient currently under anaesthesia.

What identification checks must be performed on both the patient’s wristband and the blood product
label to ensure a safe transfusion process?

a) Patient identification, including name only.


b) Patient identification, including name and date of birth only.
c) Patient identification, including name and hospital number only.
d) Patient identification, including hospital number and date of birth only.
e) Patient identification, including name, hospital number, date of birth and Social Security number.

D: Patient identification, including hospital number and date of birth only.


Since this patient is anaesthetised and unable to verbally confirm their identity, it is essential to check their
identification, including their name, date of birth and hospital number against the wristband on the patient and the
information on the blood product label. Three identifiable factors should be checked before starting a blood
transfusion by a health professional who has completed their blood transfusion training.

The blood transfusion should be commenced within 30 minutes of being out of the fridge and completed within 4
hours. Blood products should only be given through a blood administration set featuring an integral mesh filter of
170 – 200 m. This filter allows blood components to be filtered during the transfusion to remove clots and small
clumps of platelets and white blood cells that form during collection and storage. It has a smaller mesh than fluid-
giving sets, which would not remove these clumps.

A: Patient identification, including name only.


One identification factor is not enough. Numerous patients will share names, especially since these can run in
families.

B: Patient identification, including name and date of birth only.


Two identification factors are not sufficient to ensure a safe transfusion process. Although it is unlikely to be a
common event, there could be two patients in the same hospital who share both a name and a date of birth.

C: Patient identification, including name and hospital number only.


Two identifiable factors are not sufficient to ensure a safe transfusion process. Three identifiable factors need
checking before blood transfusion is begun.

E: Patient identification, including name, hospital number, date of birth and Social Security number.
The patient’s Social Security number does not need checking to ensure a safe blood transfusion.

15. A 24-year-old man undergoes blood transfusion due to intraoperative blood loss. The patient’s blood type
is A-positive (A+).

Which of the following blood products would be suitable for this patient?

a) AB-negative (AB-) blood.


b) AB-positive (AB+) blood.
c) Type A blood only.
d) Type A or type O blood.
e) Type O-ve blood only.

D: Type A or type O blood.


A patient with type A blood would have type A antigens attached to the red blood cell surface and anti-B antibodies
in plasma. They can receive type A+, A-, O+, or O- blood. The positive or negative term refers to the patient’s
rhesus D (RhD) status – 85% of the population are RhD-positive. A patient with a negative Rh status cannot receive
Rh-positive blood; a patient with a positive Rh status can receive both positive and negative blood products.

ABO blood type:


A: AB-negative (AB-)blood.
This patient has type a blood and, therefore, would have A antigens attached to the red blood cell surface and anti-B
antibodies in plasma, so he cannot receive type AB blood. He can receive type A+ , A-, O+ or O- blood.

B: AB-positive (AB+) blood.


AB+ is the universal acceptor, not donor. This means patients who are AB+ can receive any type of blood because
they have no antibodies in their plasma. The universal donor is O-negative (O-). This patient can receive type A-
positive (A+), A-negative (A-), )-positive (O+) or O-, note AB+ blood.

C: Type A blood only.


A patient with type a blood would have type A antigens attached to the red blood cell surface and anti-B antibodies
in plasma. There are no O antigens, which means they can also receive any form of type O blood.

E: Type O blood only.


O blood is the universal donor. Although this patient can receive this blood type, he can also receive group A blood
products, as well as O+ blood.

16. A 34-year-old woman who is on phentolamine while awaiting definitive treatment for phaeochromocytoma
requires emergent intubation after a road traffic accident. A decision is made to perform nasal intubation.

Which of the following drugs is the most important premedication?


a) Inhaled beta-2 agonist.
b) Intravenous (IV) dexamethasone.
c) IV hyoscine
d) Oral paracetamol
e) Topical cocaine.

E: Topical cocaine.
Phentolamine causes nasal congestion, and thus airway instrumentation has the potential to cause
significant bleeding. Topical vasoconstriction is very important in this circumstance.

A: Inhaled beta-agonist
Bleeding would be the most important consideration in this case, rather than bronchodilatation, and beta-
agonists have the potential to increase bleeding.

B: Intravenous (IV) dexamethasone.


IV dexamethasone may be useful in nasal intubation but would have the potential to increase bleeding in
this scenario.

C: IV hyoscine.
Although hyoscine can reduce the increased respiratory secretions associated with phentolamine, bleeding
is the more pressing consideration in this case, and hyoscine has the potential to increase blood loss.

E: Oral paracetamol
Analgesia may be important but is not the most important premedication in this scenario. Oral
administration would also be questionable.

17. An Anaesthetist is reviewing a patient during their intensive care attachment. The patient has sustained a
subdural haematoma following a fall. The patient is on warfarin for atrial fibrillation and has an
international normalized ration (INR) of 7.5.

Which of the following is the most important medication to administer next?

a) Factor VIII concentrate


b) Fresh frozen plasma.
c) Prothrombin – complex concentrate
d) Tranexamic acid
e) Vitamin K

C: Prothrombin – complex concentrate


In the context of active intracerebral bleeding, emergency warfarin reversal is required. Prothrombin-
complex concentrate (PCC) will reverse warfarin within minutes and is the specific antidote to warfarin
administration. Under the brand name ‘Beriplex’, PCC contains all the vitamin-K dependent clotting
factors, namely II, VII, IX ,X, protein C and protein S.
A: Factor VIII concentrate
Haemophilia is a cause of spontaneous intracerebral bleeding, but there is no suggestion of this in the stem.
Factor VIII concentrate is the product of choice for the prevention and treatment of bleeding associated
with haemophilia A.

B: Fresh frozen plasma


Fresh frozen plasma (FFP) is a leuko-depleted plasma obtained from whole blood and contains multiple
clotting factors. While FFP would reverse warfarin, both the volume required and the time it would take to
do so make it an unfavorable product for warfarin reversal.

D: Tranexamic acid
Tranexamic acid is a synthetic derivative of the amino acid lysine and binds to plasminogen. This inhibits
plasmin formation and displaces plasminogen from the fibrin surface. In turn, this stabilizes clots. It is not
indicated in this situation.

E: Vitamin K
Vitamin K should still be given in this context. However, it is not the most important next step in
medication. Due to the long half-life of warfarin, which may continue to act after PCC, vitamin K should
be given to ensure adequate reversal.

18. A 32-year-old pregnant woman presents to the Obstetric Medicine Clinic. She has a history of two
unprovoked deep vein thromboses and wants to discuss anticoagulation.

Which of the following is most appropriate for this patient?

a) Unfractionated heparin (UH)


b) Clopidogrel
c) Low molecular weight heparin (LMWH)
d) Streptokinase
e) Warfarin

Low molecular weight heparin (LMWH)


LMWHs inhibits the coagulation process by binding anti-thrombin and inhibiting factor Xa. Unlike AAT
activated by heparin, AT activated by LMWH cannot inhibit thrombin
(Factor IIa), but can only inhibit clotting factor Xa

Low molecular weight heparins are the preferred agents.

A: Unfractionated heparin (UH)


Heparin activates antithrombin III. The activated antithrombin molecule can then activate thrombin and
factor X. the size of the heparin molecules influences its binding with antithrombin III and ultimately it
anticoagulant effect.

While UH could be used for prophylaxis, it would not be first-line


B: Clopidogrel
Clopidogrel is an anti-platelet and a pro drug. Its active form irreversibly binds to the P2Y12 ADP receptor
and thereby inhibits platelet activation and aggregation by inhibiting adenosine diphosphate (ADP) uptake.

D: Streptokinase
Streptokinase forms a complex with plasminogen and so catalyses the conversion of plasminogen to
plasmin; plasmin breaks up fibrin clots. As such, streptokinase acts as a fibrinolytic and/or thrombolytic
agent. Alteplase and urokinase are other examples.

Streptokinase would be used in the case of acute, massive PE causing cardiovascular compromise, but
certainly not for prophylaxis.

E: Warfarin
Warfarin inhibits the reduction of vitamin K-dependent factors (II, VII, IX, X). there is a risk of
teratogenicity, especially in the first trimester, and should therefore be avoided.

19. Which of the following is correct regarding drugs that affect haemostatic mechanisms in the body?

a) Clopidogrel inhibits adenosine update and causes less upper GI bleeding than aspirin.
b) Dipyridamole predominantly inhibits platelets aggregation and is a coronary artery vasoconstrictor.

c) Heparin catalyses the formation of the thrombin-antithrombin complex and can induce thrombocytopenia
within 48 hours of commencement.

d) Streptokinase is an enzyme produced by Group A-beta-haemolytic streptococcoci and forms a complex


with plasminogen, catalysing the conversion of further plasminogen to plasmin.

e) Warfarin inhibits the oxidation of vitamin K and consequently reduces the production of caoagulation
factors II, VII, IX and X.

Overall Explanation
Dipyridamole predominantly inhibits platelets adhesion by inhibiting adenosine uptake.

Clopidogrel does cause less upper GI bleeding than aspirin, but acts by irreversibly preventing ADP from
binding to its receptor on the platelet surface.

Warfarin inhibits the reduction of oxidized vitamin K and streptokinase is produced by Group C beta-
haemolytic streptococci. Heparin can cause thrombocytopenia. Despite the thrombocytopenia, thrombosis
may occur and this can be potentially devastating.

20. A 30-year-old man presents to the Emergency Department following major trauma. He receivess an initial
transfusion of 4 units of O-negative packed red cells and 4 units of fresh frozenvplasma. His crossmatch
sample (sent before the administration of any blood products) reports that he is blood group AB. He
requires further blood transfusion, as part of his resuscitation and operative management.

Which of the following is most accurate regarding this patient ?


a) The paatient has A and B antigen and no antibodies.
b) The patient has A and B antigens and anti-A and anti-B antiboies.
c) The patient has A antigens and anti-B antibodies
d) The patient has neither A nor B antigens and no antibodies
e) The patient has neither A nor B antigens but has anti-A and anti-B antibodies.

A : The paatient has A and B antigen and no antibodies.


This is the correct answer. A patient who is blood group AB expresses A and B antigen on their red blood
cells. Because they express both of these , they have neither anti -A nor anti-B antibodies.

The blood group antigens and antibodies present in the ABO system are detailed here :
Blood group Antigens Antibodies
A A Anti-B
B B Anti-A
AB A&B None
O None Anti-A & Anti-B

Blood groups are commonly discussed in terms of the ABO system and the rhesus system ( eg A-positive
where ‘A’ corresponds to the ABO group and ‘positive to the rhesus status). If a patient expresses rhesus D
antigen, they are said to be positive. This part is most important in pregnancy. If a rhesus D-negative
mother carries a rhesus D-positive fetus, the mother can produce antibodies to D antigen. These antigens
can cross the placenta and cause problems if she later carries another rhesus-positive fetus.

B : The patient has A and B antigens and anti-A and anti-B antiboies.
This patient has A and B antigens on his red blood cells, and so he does not express antibodies to either of
these antigens.

C : The patient has A antigen and anti-B


This option describes a patient with blood group A.

D : The patient has neither A nor B antigen and no antibodies.


A patient with blood group AB expresses both A and B antigens. A patient who has neither of these
antigens would be blood group O, although these patients would express anti-A and anti-B antibodies.

E : The patient has neither A nor B antigens but has anti-A and anti-B antibodies.
This describes a patient with blood group O.

21. A 65-year-old man with a common femoral artery thrombus is awaiting an emergency thrombectomy. On
review of his drug chart, the Anaesthetist learns that he is receiving an unfractionated heparin (UFH)
infusion.

Which of the following best explains the reason for giving this patient an infusion of UFH?

a) Irreversible binding to antithrombin III


b) Lower molecular weigh
c) Reversible by protamine
d) Short half-life
e) Titratable effect
D: Short half-life
Unfractionated heparins (UFH) exert their anticoagulant effect by augmenting the action of antithrombin
III. Here, the main reason for giving UFH via an infusion is so that the effects can be rapidly reversed by
stopping the infusion prior to surgery. This is a function of the short half-life of UFH. Typically, the
infusion would be stopped 4-6 hours before surgery, so only a short period would exist between therapeutic
anticoagulation and the start , limiting the risk of further thrombolysis. If a more rapid reversal is required,
then protamine can be given at a dose of 1 mg (100 IU of heparin).

A: Irreversible binding to antithrombin III


Unfractionated heparin (UFH) exert their anticoagulant effect by augmenting the action of antithrombin III.
They do so in a reversible fashion.

B: Low molecular weight


Unfractionated heparin (UFH) is a heterogenous mixture of sulphated glycosaminoglycan
(mucopolysaccharide) molecules with molecular weights ranging from 3 kDa to 50 kDa. In contrast, by
subjecting UFHs to cleavage procedures, low-molecular weight heparins are produced that have an average
molecular weight of 4-5 kDa (range 1 – 10 kDa). These are excreted mainly by the kidneys and have a
longer half-life than unfractionated heparins.

C: Reversal by protamine
Protamine can be given at a dose of 1 mg (100 IU of heparin) if even more rapid reversal is required.
However, the best explanation for a giving a UFH infusion, in this case, is the short half-life, which allows
it to be stopped a few hours before surgery, limiting the risk of further thrombosis.

E: Titratable effect
While the dose of UFH is titratable and would usually be given as a bolus of 100U/kg followed by
approximately 1000 U/hour, titrated to achieve an activated partial thromboplastin time of 1.5 – 2.5 x
control, this is not the best explanation here.

22. A 77-year-old man with a past medical history of hypertension and chronic atrial fibrillation is scheduled
for an inguinal hernia repair. Two months back, the patient suffered from an acute ischaemic stroke. His
current medications include warfarin, amlodipine, rosuvastatin and Olmesartan. Physical examination
reveals a heart rate of 96/min with an irregularly irregular pulse, blood pressure 140/90 mmHg, respiratory
rate 18/min and SpO2 94% on room air. Laboratory results show an international normalized ratio (INR) of
2.5.

Which of the following is the most appropriate plan for proceeding with the surgery?

a) Delay surgery until normal sinus rhythm is restored.


b) Get cardiology consultation and proceed accordingly.
c) Proceed under general anaesthesia
d) Proceed under spinal anaesthesia.
e) Stop warfarin 5 days prior, start bridging therapy with heparin.

E: Stop warfarin 5 days prior, start bridging therapy with heparin.


Perioperative management of atrial fibrillation involves rate control and anticoagulation to mitigate the risk
of thromboembolic phenomenon. In order to proceed with the surgery under spinal anaesthesia, warfarin
must be stopped 5 days earlier. Bridging therapy with enoxaparin should be started 3 days before the
surgery. Prior to surgery, the patient’s INR should be tested. Low-dose vitamin K reversal can help patient
with an INR of 1.5 -1.9 (phytomenadione 1 – 3 mg IV). This, however, will take a few hours to take effect.
Patients should have their warfarin reintroduced as soon as possible following surgery, according to their
pre-procedure protocol. It is imperative to note that enoxaparin must be continued postoperatively together
with warfarin before the INR reaches therapeutic levels.

A: Delay surgery until normal sinus rhythm is restored.


Perioperative management of atrial fibrillation involves rate control and anticoagulation to mitigate the
risks of thromboembolic phenomon. Cardioversion to restore normal sinus rhythm is not the management
goal.

B: Get cardiology consultation and proceed accordingly.


A cardiology or haematology consult would be helpful if the surgery was to proceed in an emergency.
Since this is elective surgery, stopping warfarin and starting bridging therapy with heparin is the most
appropriate management strategy.

C: Proceed under general anaesthesia


Inguinal hernia repair is an elective procedure. Therefore, the patient must be optimized before proceeding
with the surgery. Uncontrolled atrial fibrillation can greatly increase the risk of thromboembolic events
during the perioperative period.

D: Proceed under spinal anaesthesia.


Neuraxial blockade is contraindicated in patients receiving warfarin. It can only be safely performed when
INR is <1.4

23. An 80-year-old woman in the Emergency Department is listed for emergency surgery to evacuate a lower
limb haematoma following a fall. She has a history of type II diabetes, osteoarthritis, ischaemic heart
disease and atrial fibrillation. Her medication includes ibuprofen 400 mg, co-codamol 60/500, apixaban 5
mg and bisoprolol 5 mg.

Which of the following represents the most likely contributor to the haematoma from the listed
medications?

a) Cyclo-oxygenase enzyme inhibition.


b) Direct thrombin inhibition.
c) Inhibition of the epoxide reductase enzyme.
d) Reversible inhibition of antithrombin III
e) Selective Xa inhibition.

E: Selective Xa inhibition.
Apixaban is a direct oral anticoagulant which has been prescribed to this patient as a treatment for her atrial
fibrillation. Its mechanism of action, similar to that of rivaroxaban, is to reversibly block the enzymatic
function of factor Xa in converting prothrombin to thrombin. It reaches maximum inhibition concentrations
in less than 4 hours.

A: Cyclo-oxygenase enzyme inhibition.


While the patient is on non-steroidal drug ibuprofen, a cyclo-oxygenase enzyme inhibitor, it will inhibit
platelet aggregation. This drug is less likely than the anticoagulant apixaban to be the cause of the
haematoma, though it may have been a contributing factor.

B: Direct thrombin inhibition


The direct inhibition of thrombin is the mechanism of action of the direct oral anticoagulant dabigatran.

C: Inhibition of the epoxide reductase enzyme


The inhibition of the epoxide reductase enzyme is the mechanism of action of warfarin. by inhibiting
vitamin K epoxide reductase, it inhibits the formation of gamma-carboxylated clotting factors, namely II,
VII, IX, X, protein C and S. This patient is not on warfarin.

D: Reversible inhibition of antithrombin III


This is the mechanism of action of unfractionated heparin. It accelerates the action of antithrombin III,
which is a small glycoprotein that inactivates several enzymes in the clotting cascade and hence produces
an anticoagulant effect.

24. A 67-year-old woman taking warfarin for atrial fibrillation is listed for an elective total knee replacement.
Her international normalized ratio (INR) on admission is found to be 3.2.

What is the most appropriate course of action?


a) Postpone surgery and proceed only when the INR is below 1.5.

b) Give 10 mg vitamin K, admit overnight and recheck INR the next morning. If it is below 1.5, proceed with
surgery.

c) Give prothrombin complex concentrate (PCC) and recheck INR.

d) If the surgeon is happy, proceed with surgery under general anaesthesia and tourniquet with tranexamic
acid on induction.

e) If the surgeon is happy, proceed with surgery under spinal anaesthesia and tourniquet with tranexamic acid
on induction.

A: Postpone surgery and proceed only when the INR is below 1.5.
This is an elective procedure, and therefore, it must be postponed until it is safe to proceed. Warfarin
inhibits vitamin K epoxide reductase, which reduces concentrations of vitamin-dependent clotting factors
(II, VII, IX, and X). synthesis of the natural anticoagulants, protein C and S, is also dependent on vitamin K
and is therefore reduced.

Vitamin K replacement can reduce the INR, but it takes time to take effect. Prothrombin complex
concentrate (PCC) is licensed for the reversal of over-anticoagulation, but only in an emergency situation.

B: Give 10 mg vitamin K, admit overnight and recheck INR the next morning. If it is below 1.5,
proceed with surgery.
This is not an emergency procedure, and the patient does not require preoperative admission to hospital.
The patient should be sent home with a clear plan for stopping warfarin and arranging bridging, if required.

C: Give prothrombin complex concentrate (PCC) and recheck INR.


PCC is costly and should only be given in emergency surgery. This is an elective procedure, and therefore,
giving PCC would be inappropriate.

D: If the surgeon is happy, proceed with surgery under general anaesthesia and tourniquet with
tranexamic acid on induction.
Regardless of whether the surgeon is happy, the operation should be postponed. With an INR of 3.2, blood
loss could be significant, even with the use of a tourniquet and tranexamic acid. If surgery were to proceed,
the likelihood of the patient needing an intraoperative or post-operative blood transfusion would be
unacceptably high.

E: If the surgeon is happy, proceed with surgery under spinal anaesthesia and tourniquet with
tranexamic acid on induction.
Spinal anaesthesia would be contraindicated due to the unacceptably high risk of a spinal or epidural
haematoma. Spinal anaesthesia should not be performed when the INR is above 1.4.
25. A 73-year-old man is undergoing an emergency abdominal aortic aneurysm (AAA) repair. He has a past
medical history of hypertension, gallstones and recent myocardial infarction (MI) for which he had a drug-
eluding stent implanted. His usual medications include ramipril, aspirin, clopidogrel, simvastatin and
Bendroflumethiazide.

Cross-clamping has been achieved with an estimated blood loss thus far of 2.5 litres. There have been four
units of packed red blood cells (RBCs; O -ve) and four units of fresh frozen plasma (FFP) transfused. There
has been 1 unit of packed RBCs and 1 unit of FFP given since the last full blood count (FBC)/clotting was
sent. The patient is currently haemodynamically stable, but the surgeons are struggling to achieve
haemostasis.

The latest blood results are as follows (taken after the administration of the third RBCs/FFP):

Investigation Result Normal Value


Haemoglobin (Hb) 79 g/l 135 – 175 g/l
Platelets (PLT) 87 x 109/l 150 – 400 x 109/l
International Normalized Ration (INR) 1.4 1.0
Fibrinogen 160 mg/dl 125 – 300 mg/dl

Which of the following is the most appropriate to administer next?

a) Beriplex
b) Cryoprecipitate
c) Fresh frozen plasma (FFP)
d) Packed Red Blood Cells (RBCs)
e) Platelets.

E: Platelets.
In major haemorrhages, the usual transfusion trigger would be 50 x 10 9/l. However, this patient is on dual
antiplatelet therapy (aspirin and clopidogrel) and probably has platelet dysfunction (contributing to the surgeons’
difficulty in achieving haemostasis). It would, therefore, be prudent to transfuse platelets at this stage.

A: Beriplex
Beriplex is incorrect. Beriplex should only be administered for the reversal of a vitamin K antagonist (eg warfarin).
The dose depends on the pre-treatment INR and the body weight.

B: Cryoprecipitate
Cryoprecipitate is incorrect. Cryoprecipitate is given when there is clinically significant bleeding and fibrinogen
levels are <1.5 g/l (<2g/l in obstetrics haemorrhage). Infusing two pooled units will increase fibrinogen by
approximately 1 g/l.

C: Fresh frozen plasma (FFP)


FFP is incorrect. In major haemorrhage, a ratio of at least 1 FFP:2 red cells should be achieved. Once bleeding has
been controlled, the prothrombin time (PT) ratio/INR should be below 1.5.

D: Packed Red Blood Cells (RBCs)


Packed RBCs is incorrect. While it is likely that further packed RBCs will be required, the patient is currently
haemodynamically stable and a unit has been transfused since the current blood results were taken. A further point-
of-care sample would be nice to have to guide further Hb requirements.

26. A 28-year-old woman is being prepared for an emergency lower section Caesarean section under general
anaesthesia. Ultrasound is suggestive of placenta praevia with suspicion of invasion. A radial arterial line is
placed just after induction. During the surgery, a significant amount of bleeding occurred and she was
transfused a total of 8 units of packed red blood cells (RBCs). The estimated blood loss is around 2.5 litres.

Which of the following is the most appropriate next step in the management of this patient?

a) Give furosemide 40 mg intravenously (IV).


b) Give vitamin K
c) Transfuse 8 units of factor VII concentrate
d) Transfuse platelets and fresh frozen plasma (FFP)
e) Transfuse 8 units of prothrombin complex concentrate.

D: Transfuse platelets and fresh frozen plasma (FFP)


Coagulopathy is linked to haemoglobin (blood loss) and blood product transfusions (dilution). Therefore, it
is recommended to replace the blood loss in a major haemorrhage by transfusing red blood cells (RBCs):
fresh froze plasma (FFP): Platelets in a 1: ! : 1 ratio.

A: Give furosemide 40 mg intravenously (IV).


A 5 – 10 mg dose of furosemide will be enough to offset the effect of increased ADH due to painful
surgical stimulation. Moreover, as long as the urine output is adequate and there are no signs of fluid
overload, there is no need to give furosemide.

B: Give vitamin K.
Vitamin K is administered along with FFP in patients taking warfarin and presenting with major bleed or
needing urgent surgery.

C: Transfuse 8 units of factor VII concentrate


The US Food and Drug Administration (FDA) has only approved recombinant factor VIIa (rFVIIa) for the
prevention and treatment of bleeding episodes in patients with congenital factor VII deficiency. The role of
factor VII concentrate in life-threatening haemorrhage is an off-label indication backed by limited
evidence.

E: Transfuse 8 units of prothrombin complex concentrate.


Prothrombin complex concentrates cannot be used as a monotherapy as it is deficient in factor V. when
used as an adjunct to FFP during coagulopathy, it helps to correct the international normalized ratio (INR)
faster.

27. A 78-year-old man with Corona virus disease 2019 pneumonitis develops multiple small pulmonary emboli
while in the Intensive Care Unit. In discussion with Haematology, the team is advised to commence
anticoagulation. The patient’s anticoagulant effect is monitored using activated partial thromboplastin time
(aPTT) levels.

Which of the following drugs was most probably used in this patient?

a) Apixaban
b) Lower-molecular-weight heparin
c) Rivaroxaban
d) Unfractionated heparin.
e) Warfarin.

D: Unfractionated heparin.
Unfractionated heparin works by activating antithrombin, thereby preventing prothrombin from converting
to thrombin. For this reason, it can be monitored via aPTT levels. It can be employed in the treatment of
heart attacks and unstable angina, but should be employed cautiously in patients with poor renal function.
The side effects of unfractionated heparin include a decreased platelet count, injection site pain and
increased bleeding, as well as heparin-induced thrombocytopenia.

A: Apixaban
Apixaban is a Factor Xa inhibitor and so cannot be monitored via aPTT levels.

B: Low-molecular weight heparin


Low-molecular weight heparin is a Factor Xa inhibitor and so cannot be monitored via aPTT levels.
However, although not routinely done, it can be monitored via anti-Xa levels.

C: Rivaroxaban
Rivaroxaban is a factor Xa inhibitor, meaning that it cannot be monitored via aPTT levels.

E: Warfarin
Warfarin is a coumarin derivative, a vitamin K epoxide-reductase inhibitor, and is monitored via
international normalized ratio levels, not aPTT.

28. A 83-year-old man comes to the General Practitioner because his latest three INR readings have been 1.5 –
1.7, distinct from his usual range between 2 and 3 . he has a history of hypertension and a mechanical heart
valve. The patient takes amlodipine 10 mg, metoprolol 50 mg and warfarin 5 mg daily. He recently started
taking the herbal supplement ginseng to boost his energy levels.

Which of the following best explains the reason for this patient’s failure to achieve the INr target?

a) Interference of metoprolol with the action of warfarin


b) Reduced plasma concentration of warfarin by ginseng.
c) Reduced plasma concentration of warfarin due to amlodipine competing for binding sites.
d) Reduced protein binding of warfarin due to ginseng.
e) Suboptimal dosing of warfarin.

B: Reduced plasma concentration of warfarin by ginseng.


The herbal supplement ginseng has been found to reduce the plasma levels of warfarin and thus decrease
the INR.

A: Interference of metoprolol with the action of warfarin.


Metoprolol and warfarin can be safely taken with no adverse drug interaction.
C: Reduced plasma concentration of warfarin due to amlodipine competing for binding sites.
Warfarin is highly protein-bound in plasma. Consequently, drugs that are highly protein-bound, such as
amlodipine, can displace warfarin from protein-binding sites and increase the INR and risk of bleeding.

D: Reduced protein binding of warfarin due to ginseng.


Reduced protein binding would result in an increase in the plasma concentration of unbound drug and, as a
result, increased warfarin at binding sites, with increased INR and in increased risk of bleeding.

E: Suboptimal dosing of warfarin.


The patient was previously attaining his target INRs with the current dose, so that is unlikely to be the
reason for his recent subtherapeutic INRs.

29. A 66-year-old man is scheduled for an open abdominal aortic aneurysm repair under general anaesthesia.
The Outpatient Clinic assessment revealed that his medications include atenolol, atorvastatin, warfarin and
ramipril. During the operation, there is significant blood loss, but a good recovery is made.
Thromboelastography (TEG) shows a normal reaction time (R value) with a decrease maximum amplitude
(MA).

Which of the following medications is most likely to have contributed to the pattern described?

a) Atenolol
b) Ginseng
c) Heparin (given intraoperatively)
d) Hydroxyethyl starch
e) Warfarin

B: Ginseng
Care should be taken when gathering an anaesthetic history to include any inhalers, injections, drugs of
abuse and herbal remedies. Many herbal remedies have anticoagulant properties, as well as significant
drug-drug interactions. Ginseng acts as an antiplatelets, which is consistent with the TEG findings.
Platelets, as well as fibrinogen, are require to achieve maximum amplitude.

A: Atenolol
In the laboratory, at least, atenolol causes platelets aggregation, hence a theoretical increase in thrombotic
risk, which is at odds with the TEG results.

C: Heparin (given intraoperatively)


Heparin would be expected to cause a prolonged R value with or without a decrease in maximum
amplitude.

D: Hydroxyethyl starch
Hydroxyethyl starch is no longer used in clinical practice in the United Kingdom. It causes abnormal
platelet aggregation. It impairs platelets reactivity by causing a reduction in the availability of the platelet
fibrinogen receptor glycoprotein IIb/IIIa and also by decreasing the platelet -linking properties of Von
Willebrand factor.

E: Warfarin
As warfarin affects clotting factors, a prolonged R value would have been expected, which is not seen here.
Typically, warfarin is stopped in the days before surgery or reversed activity in cases of emergencies.

30. A 64-year-old man with a lung nodule in the right lower lobe is scheduled for a video-assisted thoracic
surgery (VATS) wedge resection. His past medical history is significant for diabetes mellitus and ischaemic
heart disease. He underwent percutaneous coronary stenting six months ago. A recent echocardiogram
revealed an ejection fraction of 55 %. His current medications include sitagliptin 50 mg, metformin 500
mg, aspirin 75 mg, clopidogrel 75 mg and rosuvastan 10 mg. Thoracic epidural is planned for postoperative
pain management.
Which of the following is the most appropriate recommendation to make during the preoperative
evaluation of this patient?

a) Proceed with surgery and epidural placement provided INR <1.5


b) Skip sitagliptin and metformin on the day of surgery.
c) Stop aspirin three days prior to surgery.
d) Stop clopidogrel 1 week prior to surgery.
e) Stop oral hypoglycaemics and shift the patient to Variable Rate Intravenous Insulin infusion.

D: Stop clopidogrel 1 week prior to surgery.


Association of Anaesthetist of Great Britain and Ireland (AAGBI) guidelines recommend stopping
clopidogrel at least 1 week prior to surgery. Likewise, the American Society of Regional Anaesthesia
(ASRA) recommends a 7-day gap between stopping clopidogrel and doing a neuraxial procedure.
Antiplatelet agents such as clopidogrel increase the chance of bleeding, which could cause complications
such as spinal haematoma following neuraxial blockade.

A: Proceed with surgery and epidural placement provided INR <1.5


Clopidogrel increases perioperative bleeding, so surgery is often delayed while treatment is discontinued.
Since this is an elective procedure, the patient should be optimized before proceeding with surgery.

B: Skip sitagliptin and metformin on the day of surgery.


AAGBI guidelines allow morning dose of metformin and dipeptidyl peptidase IV(DPP-IV) inhibitors to be
taken as routine on the morning of surgery.

C: Stop aspirin 3 days prior to surgery.


Current guidelines allow low-dose aspirin to be continued up until the day of surgery.

E: Stop oral hypoglycaemics and shift the patient to Variable Rate Intravenous Insulin infusion.
AAGBI guidelines allow morning dose of metformin and dipeptidyl peptidase IV (DPP-IV) inhibitors to be
taken in routine on the morning of surgery.
31. A 54-year-old man underwent pancreaticoduodenectomy (Whipple’s procedure) under general anaesthesia.
A thoracic epidural was placed between T8 and T9 for postoperative pain management. Following the
surgery, the patient was moved to surgical Intensive Care Unit. The patient received enoxaparin for deep
vein thrombosis prophylaxis. A weaning trial was planned on the 3 rd postoperative day after ensuring all the
parameters were optimized.

Which of the following is the most appropriate course of action with regard to removing the epidural
catheter from this patient?

a) Remove epidural catheter 4 hours after the last dose of enoxaparin.


b) Remove epidural catheter 8 hours after the last dose of enoxaparin.
c) Remove epidural catheter 12 hours after the last dose of enoxaparin.
d) Remove epidural catheter 24 hours after the last dose of enoxaparin.
e) Remove epidural catheter provided the INR < 1.5

C: Remove epidural catheter 12 hours after the last dose of enoxaparin.


AAGBI 2013 guidelines and American Society of Regional Anaesthesia and Pain Medicine (ASRA)
guidelines recommend that in patients receiving low molecular weight heparin (LMWH) for
thromboprophylaxis, the epidural catheter should be removed 12 hours after the last dose of LMWH such
as enoxaparin. Subsequent LMWH dosing should occur at least 4 hours after catheter removal.

A: Remove epidural catheter 4 hours after the last dose of enoxaparin


A 4-hour delay following epidural catheter removal before LMWH is resumed has been recommended for
patients receiving higher therapeutic doses of enoxaparin (I mg/kg bid or 1.5 mg/kg once daily.

B: Remove epidural catheter 8 hours after the last dose of enoxaparin.


Current guidelines recommend a range of at least 12 hours following the last dose of LMWH for both
needle/catheter placement and catheter removal.

D: Remove epidural catheter 24 hours after the last dose of enoxaparin.


If the patient is on a therapeutic dose of LMWH, the catheter should be removed 24 hours after the last
dose of enoxaparin.

E: Remove epidural catheter provided the INR < 1.5


American Society of Regional Anaesthesia and Pain Medicine (ASRA) guidelines have clearly defined a
set time period of 12 hours after the last dose of LMWH for the safe removal of the epidural catheter.
Currently, there is no such recommendation to base this decision on the INR of the patient.

32. A 29-year-old woman with von Willebrand’s disease (vWD) is scheduled for Caesarean section under
general anaesthesia. The patient has haematomas on both arms and legs and a history of abdominal wall
haematoma in a previous Caesarean section.

Which of the following will NOT be helpful in achieving haemostasis during the surgery?

a) Cryoprecipitate.
b) Desmopressin
c) Factor III concentrate
d) Fresh frozen plasma (FFP)
e) Recombinant factor VII

E: Recombinant factor VII


Recombinant factor VII (rFVII) will not be helpful in achieving haemostasis in this patient with von
Willebrand’s disease (vWD) as it does not contain von Willebrand’s factor (vWF). rFVIIa has been
approved by the US Food and Drug Administration for use in Glanzmann’s thrombasthenia and factor VII
deficiency.

A: Cryoprecipitate
Patients with vWD have prolonged bleeding times and a reduced amount of factor VIII. Cryoprecipitate
will be helpful in achieving haemostasis in this patient as it contains both vWF and factor III.

B: Desmopressin
The most common hereditary abnormality affecting platelet functions is vWD, which is caused by a
quantitative or qualitative deficiency of a protein known as von Willebrand’s factor (vWF). Desmopressin
is used to treat vWD because it increases the release of available vWF.

C: Factor III concentrate


Factor III or tissue factor is a potent stimulator of vWF secretion by endotracheal cells. Therefore, it can
also be helpful in patients with vWD.

D: Fresh frozen plasma (FFP)


vWF is produced by endothelia cells and platelets. Patients with vWD have prolonged bleeding time and a
reduced amount of factor VIII. FFP will be helpful in this patient if it contains vWF.

33. A neonate born at term weighing 3 kg is undergoing transverse loop colostomy for high type imperforate
anus on day 5 of life. During the surgery, the patient starts bleeding actively.

What is the estimated blood volume of this neonate?

a) 220 ml
b) 230 ml
c) 255 ml
d) 290 ml
e) 340 ml

C: 255 ml
The estimated blood volume of a term neonate is around 80 -90 ml/kg. this approximates to somewhere
between 240 – 270 ml. Hence the value of 255 ml is most appropriate.
34. A 34-year-old man is rushed to the Emergency Department following a gunshot wound to the abdomen. He
is bleeding profusely. his vitals are a heart rate of 120/min, blood pressure of 90/50 mmHg, SpO 2 98% on
room air and respiratory rate 27/min. The radial pulse is feeble and the patient is responding vaguely. IV
access line is achieved. Meanwhile, a massive transfusion protocol is activated.

Which of the following is the resuscitation fluid of choice for this patient?

a) Plasma, packed red blood cell (pRBSs) and platelets in a 1:1:1 ration.
b) Type O, Rh-ve pRBCs
c) Type – specific, fresh whole blood.
d) Plasma and pRBCs in 1:1 ratio.
e) Cold-stored low-titre O whole blood.

D: Plasma and pRBCs in 1:1 ratio.


Plasma and pRBCs in a 1:1 ratio is the preferred resuscitation fluid according to the latest British Society
for Haematology (BSH) guidelines. In traumatic haemorrhage, the early transfusion of fresh frozen plasma
in a fixed ratio to red cells is recommended to reverse coagulopathy and prevent bleeding. This ratio is 1: 2
in a non-traumatic setting.

A: Plasma, packed red blood cells (pRBCs) and platelets in a 1:1:1 ration.
It is imperative to assess for haemorrhagic shock (altered mental status in the absence of brain injury and/or
weak or absent radial pulse) and activate massive transfusion protocol early on. Dilutional coagulopathy is
caused by the transfusion of large amounts of red cells and other intravenous fluids that lack coagulation
factors or platelets. Therefore, in addition to packed red blood cells (pRBCs), there is need to transfuse
fresh frozen plasma (FFP) and platelets. Until 1-5 – 2.5 blood volumes have been replaced, the platelet
count normally remains above 50,000 /l. Since a lot of hospitals do not have platelets on site, BSH
guidelines recommend transfusion of pRBCs and FFP only, in a 1:1 ratio.

B: Type O, Rh-ve pRBCs


Packed red blood cells alone are never the resuscitation fluid of choice. They must be supplemented by FFP
due to the dilutional coagulopathy resulting from the transfusion of large amounts of red cells and other
intravenous fluids that lack coagulation factors or platelets.

C: Type – specific, fresh whole blood.


Transfusion of unscreened group O fresh whole blood or type-specific fresh whole blood can only be done
under proper medical supervision by qualified staff due to the increased risk of a potentially fata
haemolytic reaction.

E: Cold-stored low-titre O whole blood.


Cold-stored low-titre O whole blood is usually not available outside a military setting. It is preferred
resuscitation fluid for patients in haemorrhagic shock according to military guidelines: Joint Trauma
System/Tactical Combat Casualty Care (JTS/TCCC)

References:
Bristish Society for Haematology Guideline: Haematological management of major heamorrhagePDF
Screening Donated Blood.
This is to make sure that patients receive blood that matches their
blood type. Before transfusion, the donor and blood unit are also
tested for certain additional proteins (antibodies) that may cause
adverse reactions in a person receiving a blood transfusion.
The use of low-titer group O whole blood (LTOWB) in military and civilian trauma
centers shows no significant difference in outcomes compared with component therapy.

Conclusions.—: LTOWB therapy appears no worse than using standard component


therapy in nontrauma patients requiring a massive transfusion activation, suggesting
that LTOWB is a reasonable alternative to component therapy in nontrauma, civilian
hospital patients, even when blood type is known.

The rebirth of the cool: a narrative review of


the clinical outcomes of cold stored low titer
group O whole blood recipients compared to
conventional component recipients in trauma
There has been renewed interest in the use of low titer group O whole blood (LTOWB)
for the resuscitation of civilian casualties. LTOWB offers several advantages over
conventional components such as providing balanced resuscitation in one bag that
contains less additive/preservative solution than an equivalent volume of conventional
components, is easier and faster to transfuse than multiple components, avoids blood
product ratio confusion, contains cold stored platelets, and reduces donor exposures.
The resurgence in its use in the resuscitation of civilian trauma patients has led to the
publication of an increasing number of studies on its use, primarily amongst adult
recipients but also in pediatric patients. These studies have indicated that hemolysis
does not occur amongst adult and pediatric non-group O recipients of a modest
quantity of LTOWB. The published studies to date on mortality have shown conflicting
results with some demonstrating a reduction following LTOWB transfusion while most
others have not shown a reduction; there have not been any studies to date that have
found significantly increased overall mortality amongst LTOWB recipients. Similarly,
when other clinical outcomes, such as venous thromboembolism, sepsis, hospital or
intensive care unit lengths of stay are evaluated, LTOWB recipients have not
demonstrated worse outcomes compared to conventional component recipients. While
definitive proof of the trends in these morbidity and mortality outcomes awaits
confirmation in randomized controlled trials, the evidence to date indicates the safety of
transfusing LTOWB to injured civilians.

35. A 36-year-old man is rushed to the Emergency Department following blunt trauma to the abdomen. His
vitals are blood pressure (BP) 90/60 mmHg, heart rate 123/min, temperature 37 o C and respiratory rate
22/min/ focused assessment with sonography in trauma scan shows free fluid in the abdomen. X-ray trauma
series including chest X-ray (CXR), upper limbs, lower limbs, pelvis, and head and neck are done. A blood
sample is taken for blood caeruloplasmin, grouping and cross-match. The decision is made for urgent
exploration. Blood transfusion is started. Half an hour later, the patient suddenly becomes short of breath.
His temperature is 37.4 o C, BP is 100/60 mmHg, pulse is 111/min, respiratory rate is 28/min and oxygen
saturation is 88% on room air. Repeat CXR shows bilateral pulmonary infiltrates.

Which of the following represents the pathogenetic mechanism behind the most likely diagnosis in
this case?

a) Trauma to lung tissue and capillaries.


b) Transfusion-associated circulatory overload.
c) Pericardial fluid accumulation.
d) Antibody binding of leukocytes.
e) Antibody binding of red blood cells.

D: Antibody binding of leukocytes.


Transfusion-related acute lung injury (TRALI) is a typical transfusion reaction that manifests as shortness
of breath and pulmonary oedema on radiograph 30 minutes after the transfusion is started. In TRALI, the
capillary leak in the pulmonary vasculature is due to the donor antibodies reacting against recipient
leukocytes. TRLI is treated with support therapy since it resolves on its own. Diuretics have no role in
TRALI because the underlying disease is microvascular injury rather than fluid overload.

A: Trauma to lung tissue and capillaries.


Pulmonary contusion is described as a traumatic injury to the lung tissue and capillaries. Pulmonary
contusion presents in the same way as transfusion-related acute lung injury TRALI) and has the same chest
x-ray. TRALI is a more likely diagnosis in this case because of the time period of the patient’s presentation
and the correlation with transfusion.

B: Transfusion-associated circulatory overload.


Transfusion-associated circulatory overload is caused by the inability of the recipient to compensate for the
transfused volume. It is usually seen following massive transfusion.

C: Pericardial fluid accumulation


Pericardial fluid accumulation is unlikely as it would present with findings of cardiac tamponade.

E: Antibody binding of red blood cells


Antibody binding of red blood cell antigens represents ABO incompatibility. In such a case, the patient will
experience signs of haemolysis, including shortness of breath and dark urine.
36. A 65-year-old woman with a history of well-controlled epilepsy is admitted to hospital with a seizure
thought to have been triggered by a chest infection. Levetiracetam is administered which terminates the
seizure. She also developed new-onset atrial fibrillation, which was rate-controlled. She was sent home on
warfarin, as well as her usual medications. Two weeks later, her INR is still below the therapeutic range
despite significant doses of warfarin.

Which of this patient’s medications is most likely to be responsible for the derangement of her INR?

a) Clonazepam.
b) Lamotrigine
c) Levetiracetam
d) Pregabalin
e) Topiramate

E: Topiramate
Topiramate antagonizes the effects of the aminomethylphosphonic acid receptor on the post-synaptic
excitatory neurone, thereby inhibiting transmission. It is also thought to work on voltage-gated sodium
channels, further inhibiting GABA transmission, with some non-specific effects at the N-methyl-D-
aspartate receptor. Along with more familiar inducers such as carbamazepine, phenobarbital and phenytoin,
topiramate induces the cytochrome P450 system and increases the metabolism of warfarin, hence reducing
the plasma concentration available and, therefore, the INR.

A: Clonazepam.
Clonazepam is a benzodiazepine which binds to a specific site on the gamma-aminobutyric acid (GABA)
receptor, which is a ligand-gated, chloride ion-selective channel. When it binds to the receptor, it increases
the frequency of chloride channel opening. This results in hyperpolarization and reduced firing of the
neurones, as well as the inhibition of transmission of the synapse. Note that it is affected by other
medications that alter the cytochrome P450 system.

B: Lamotrigine.
Lamotrigine is an antiepileptic used in the treatment of focal seizures. It is thought to work by inhibiting
voltage-gated sodium channels, thereby preventing the depolarization of neurones. It does not have any
significant interactions with warfarin.

C: Levetiracetam
Levetiracetam works on the synaptic vesicle glycoprotein 2A anchor, holding glutamate vesicles in place
on presynaptic excitatory neurones. It is not inducer as it prevents release and transmission.

D: Pregabalin
Pregabalin, best known for its use in neuropathic pain, is also an antiepileptic that works by binding to the
alpha-2 -delta subunit of voltage-gated calcium channels in presynaptic excitatory neurones. Thus, it
prevents the vesicular release and transmission of glutamate.
37. A 67-year-old woman is brought to the operating theatre with a suspected ruptured aortic aneurysm. The
major haemorrhage protocol is activated, and, after 5 units of red blood cells and 5 units of fresh frozen
plasma (FFP) , the Anaesthetist decides to give a pool of platelets.

Which of the following is the most appropriate storage temperature for platelets?

a) -15 o C
b) – 12 o C
c) 1–4oC
d) 2–5oC
e) 20 – 24 o C

E: 20 – 24 o C
Platelets are stored at around room temperature under constant agitation. In instances of a major
haemorrhage, platelets are usually given either after 5 units of red blood cells and 5 units of FFP, as one
pool of platelets is made up of 10 units of blood or when the patient’s platelets count is below 75 x 10 9/L.

A: -15 o C
Plasma is stored at a temperature of -15 o C, not platelets.

B: – 12 o C
Neither red blood cells, plasma nor platelets are stored at this temperature. Platelets are stored at around
room temperature under constant agitation, and red blood cells at 1 – 4 o C.

C: 1 – 4 o C
Whole blood is separated into its constituent parts of red blood cells, plasma and platelets via
centrifugation. This is the temperature red blood cells are stored at, not platelets.

D: 2 – 5 o C
2 – 5 o C is not the temperature at which platelets are stored. Platelets are stored at around room
temperature under constant agitation.

38. An anaesthetist is reviewing a patient on the ward prior to surgery for a hemiarthroplasty. The patient, 24
hours ago, received their last treatment dose of rivaroxaban 20 mg for a pulmonary embolism. The patient
has a background of hypertension, pulmonary embolism and atrial fibrillation. The Anaesthetist calculates
the patient’s creatinine clearance (CrCL) to be 55 ml/minute.

When is the earliest time from now that spinal anaesthesia could be attempted in this patient?

a) 18 hours.
b) 24 hours
c) 48 hours
d) 72 hours
e) Immediately.
B: 24 hours
The Association of Anaesthetists of Great Britain and Ireland (AAGBI) guidelines recommend for a patient
with a CrCl > 30 ml/minute, a minimum period of 48 hours from a treatment dose of rivaroxaban to
recommended time of performance of spinal anaesthesia. As such, it is recommended that a further 24
hours elapses until it is safe to perform blockade in this patient. It would not be deemed safe to perform a
spinal block immediately.

This is based in most circumstances on time to peak drug effect + (elimination half-life x 2), after which
time less than a quarter of the peak drug level will be present. It is worth noting that another factor here
would be balancing the surgical risk itself with the patient having taken a direct oral anticoagulant
(DOAC). Generally, for a procedure such as this, with a high risk of bleeding, as omission of a DOAC 49
hours prior and 48 hours following would be recommended (in the elective setting), based on the
Perioperative Anticoagulation use for Surgery Evaluation study.

A: 18 hours.
After a prophylactic dose of rivaroxaban (10 mg once daily), a spinal anaesthetic can safely be performed
after 18 hours. However, this would mean the spinal could have been done anytime from 6 hours
previously. The case states the patient is taking treatment-dose rivaroxaban (ie 20 mg once daily), so it is
usually recommended a total of 48 hours must have elapsed prior to a spinal block.

C: 48 hours
This is the recommended length of time from rivaroxaban dose to spinal blockade. However, since the dose
was given 24 hours ago, a further 24 hours should elapse until a spinal block is possible.

D: 72 hours.
Another DOAC, dabigatran, typically requires longer from dose to safe recommendation of a spinal
blockade. Dependent on CrCl, a minimum of 96 hours may be required before spinal blockade can be
recommended.

E: Immediately
The AAGBI guideline recommend for a patient with a CrCl > 30 ml/minute, a minimum period of 48 hours
from a treatment dose of rivaroxaban to the recommended time of performance of spinal anaesthesia. As
such, it is recommended that a further 24 hours elapse until it is safe to perform blockade in this patient. It
would not be deemed safe to perform a spinal block immediately.

39. An Anaesthetist is in the Pre-assessment Clinic with their Consultant and is asked to review a set of notes
for a patient listed for an elective total knee replacement next week. The patient has had a recent
myocardial infarction and underwent primary percutaneous coronary intervention 4 weeks ago. The
patient’s current medications include aspirin, bisoprolol, atorvastatin, amlodipine, clopidogrel and
sertraline.

Which of the following is the best perioperative management plan regarding the antiplatelet
therapy?

a) Continue clopidogrel, stop aspirin.


b) Continue with tirofiban bridging.
c) Delay surgery.
d) No action required.
e) Stop clopidogrel for 7 days, continue aspirin.
C: Delay surgery.
This patient is in a very high-risk category for major adverse cardiac event (MACEs) in the perioperative
period and is presenting for elective surgery. Elective surgery in patients at high thrombotic risk should be
deferred until they are at a lower level of risk. In all cases, a careful liaison between surgeons, cardiologists,
heamatologists and anaesthetist is required when managing patients on dual antiplatelets therapy who are
attending for surgery. This will include weighing up the thrombosis risk, risk of MACE and the urgency
and bleeding risk associated with the surgery required.

A: Continue clopidogrel, stop aspirin.


Clopidogrel inhibits adenosine-diphosphate-receptor-mediated platelet aggregation via the P2Y 12 receptor.
Its effects are permanent and irreversible for the duration of the platelet lifespan (7 – 10 days). It is unlikely
clopidogrel would be continued and aspirin stopped.

B: Continue with tirofiban bridging.


Tirofiban is a glycoprotein IIa/IIb-receptor antagonist that prevents the cross-linking of platelets, the final
common pathway of platelets action. In certain circumstances, its continued bridging might be considered
in this context, but this would be specialist advice from cardiologist and haematologists.

D: No action required.
In patients with a recent acute coronary syndrome or coronary artery stent on dual antiplatelets therapy,
urgent surgery with a low bleeding risk can proceed without the interruption of antiplatelet therapy.
However, the orthopaedic procedure would not be deemed at low risk of bleeding.

E: Stop clopidogrel for seven days, continue aspirin.


Aspirin exerts its antiplatelet effects primarily via cyclo-oxygenase -1 enzyme inhibition, reducing
thromboxane A2 production. In the context of specialist perioperative advice, aspirin needs to be continued,
and clopidogrel stopped.

40. A 77-year-old man presents to the Emergency Department with haematemesis. He is in hypovolaemic
shock, with a blood pressure of 70/47 mmHg and a pulse of 130 bpm. A blood gas analysis has revealed a
heamoglobin level of 68 g/l (Normal value: 135 – 175 g/l). he is being transfused with red blood cells.
During the transfusion, he complains of dyspnoea, feeling hot and a headache. His temperature is 38.5 o C.

Which of the following best explains this patient’s symptoms?

a) Acute non-haemolytic febrile reaction.


b) Anaphylaxis.
c) Hepatitis resulting from contaminated transfused blood.
d) Normal reaction to blood transfusion.
e) Transfusion-associated lung injury.

A: Acute non-haemolytic febrile reaction.


The symptoms of an acute non-haemolytic febrile reaction include pyrexia, chills, rigours and headache.
The condition’s management includes:
 Immediate recognition
 Stopping blood transfusion.
 A-E assessment
 Informing the team and the senior.
 Doubling the IV fluids being transfused/giving an IV fluid bolus and
 Treating the temperature rise with paracetamol
If on obvious concerns exist and blood is still required, restarting at a slower rate while closely monitoring
the evolving situation should be considered.

B: Anaphylaxis.
Tachycardia and hypotension can be due to anaphylaxis. As per National Audit Project reports,
hypotension is one of the first signs of anaphylaxis in an anaesthetised patient. However, in this case, when
taking into consideration the history of vomiting and the significant anaemia, hypovolaemia seems to be a
more likely cause.

C: Hepatitis resulting from contaminated transfused blood.


The transmission of pathogens can occur during a blood transfusion. However, there is very low risk due to
the rigorous processes that are currently undertaken at all stages of acquiring and storing blood. Hepatitis is
unlikely to present this early on, and the symptoms and signs do not fit with this diagnosis.

D: Normal reaction to blood transfusion.


This in not a normal reaction to a blood transfusion. A blood transfusion should not cause fever or
hypotension. In many cases, a blood transfusion would increase the patient’s blood pressure.

E: Transfusion-associated lung injury.


A transfusion-associated lung injury is a known side-effect of blood transfusion. There is a rapid onset of
lung injury and non-cardiogenic pulmonary oedema. These signs are absent in this case.

41. During a shift in the Intensive Care Unit, an Anaesthetist is awaiting a major trauma arrival. The patient has
received tranexamic acid 1 g prior to arrival at the Emergency Department.

Which of the following best describes the mechanism of action of tranexamic acid?

a) Augmenting antithrombin III.


b) Conversion of plasminogen to plasmin.
c) Inhibition of plasmin formation.
d) Mimicking tissue plasminogen activator.
e) Promotion of fibrinolysis.

C: Inhibition of plasmin formation.


Fibrinolysis is a key component of the vascular system that helps ensure vessel patency. This process starts
with plasminogen being broken down to plasmin by tPA, which is in turn, causes fibrin to break down into
fibrin degradation products. Fibrin is a key component of clots; hence, fibrinolysis breaks down clots.
Tranexamic acid is a synthetic derivative of the amino acid lysine and binds to plasminogen. This inhibits
plasmin formation and displaces plasminogen from the fibrin surface. In turn, this stabilizes clots. The
Clinical Randomization of an Antifibrinolytic in Significant Haemorrhage-2 report showed a small absolute
risk reduction in mortality with the use of tranexamic acid in major trauma, though tranexamic acid is used
widely in many surgeries with active or potential bleeding.
Mechanism of action of TXA. Profibrinolytic effects are in green, and antifibrinolytic effects are in red. The predominant
mediator of postinjury hyperfibrinolysis appears to be tPA released from ischemic endothelium. PAI-1 binding of circulating tPA
appears to be a major mechanism for the postinjury shutdown. 8,9 In the presence of high uPA, TXA might increase
plasminogen-mediated fibrinolysis and bleeding. 10,11 aPC indicates activated protein C; PAI 1/2, plasminogen activator
inhibitor 1 (endothelium) and 2 (placenta); TAFI, thrombin activatable fibrinolysis inhibitor; tPA, tissue plasminogen activator;
TXA, tranexamic acid; uPA, urokinase-type plasminogen activator. Modified from Lier and Maegele, Blutungsmanagement:
tranexamsäure in der präklinik. Pro und kontra [Coagulation management: prehospital tranexamic acid. Pro and contra].
Notfall+Rettungsmedizin, 2018, with permission. 12

A: Augmenting antithrombin III


Drugs that augment antithrombin III activity (such as heparin) act to inhibit thrombin and are anticoagulant drugs.

B: Conversion of plasminogen to plasmin.


This is the role of a tissue plasminogen activator (tPA). Tranexamic acid inhibits this process.
D: Mimicking tissue plasminogen activator.
Mimicry of a tPA is a mechanism of action of alteplase, which is a recombinant tPA designed to break down blood
clots.

E: Promotion of fibrinolysis
Tranexamic acid inhibits excessive fibrinolysis. This reduces the speed of clot breakdown.
42. An Anaesthetist is managing a major obstetric haemorrhage during a category 1 caesarean section. There is
ongoing bleeding suspected and 2 units of group-specific blood, 1 g of tranexamic acid and 2 litre of
Hartmann’s solution have been given thus far. Investigations are shown.

Investigation Result Normal Value


Haemoglobin (Hb) 79 g/l 115 – 155 g/l (Female)
Platelets (PLT) 105 x 109/l 150 – 400 x 109/l
Activated Partial Thromboplastin Time ratio (APTr) 1.3 0.8 – 1.2
Ionised calcium 1.2 mmol/l 1.2 – 1.4 mmol/l
fibrinogen 100 mg/dl 125 – 300 mg/dl

Which of the following is the most important to administer next?

a) 1 unit O-ve blood


b) Cryoprecipitate
c) Fresh frozen plasma
d) Prothrombin-complex concentrate.
e) Two pools platelets.

B: Cryoprecipitate
This patient has hypofibrinogenaemia, which is associated with the severity of major obstetric
haemorrhage. An initially low fibrinogen level is associated with more significant blood loss. A fibrinogen
concentration < 2 g/dl is shown to have a 100 % positive predictive value for severe postpartum
haemorrhage. Cryoprecipitate contains around 15 g/l.
It has a higher concentration of fibrinogen and a lower infusion volume than fresh frozen plasma (FFP), but
still requires thawing. Fibrinogen concentrate is being developed, which overcome many of the drawbacks
of cryoprecipitate, although it is not in routine use in the United Kingdom at present.

A: 1 unit O-ve blood


The question states that the patient has already been given group-specific blood. While they are likely to
have an ongoing packed-red cell requirement in the context of a falling Gb and ongoing active bleeding, the
use of non-group- specific blood when Anaesthetist knows group-specific blood is available is redundant.

C: Fresh Frozen Plasma


Fresh frozen plasma is leuko-depleted plasma obtained from whole blood and contains multiple clotting
factors. It is indicated empirically in trauma [(at least in a 1:2 ratio of FFP: Packed Red Blood cells) –
FFB:pRBCs = 1:2] and in patients with multiple clotting factors deficiencies, such as disseminated
intravascular coagulation. In this case, it would be indicated if the activated partial thromboplastin time
and/or prothrombin time ratio were > 1.5. FFP contains a small amount of fibrinogen but is less effective
than cryoprecipitate for replacing fibrinogen and has numerous drawbacks in this setting, including the
need for a large infusion volume.
D: Prothrombin-complex concentrate
Prothrombin-complex concentrate (PCC) contains factors II, VII, IXX and X. it has replaced FFP as the
recommended treatment for rapid warfarin reversal. Bleeding due to the coagulopathy associated with liver
disease is another instance where PCC may also be used in treatment.

E: Two pools platelets.


Platelets transfusion may be required in this case, but with a platelet count > 100 x 10 9/l, it is not the most
appropriate next step in treatment in this case.
43. A 50-year-old man is admitted to an Intensive Therapy Unit (ITU) following an emergency ruptured
abdominal aortic aneurysm repair. He has received 6 units of blood and 4 units of fresh frozen plasma plus
5 litres of crystalloid.
The abdominal dressing is heavily bloodstained, and he is cardiovascularly stable.
His blood results on admission to the ITU are as follows:

Investigation Result Normal Value


Haemoglobin (Hb) 81 g/L 135 – 175 g/L
Platelets 82 x 109/L 150 – 400 x 109/L
International Normalized Ratio (INR) 1.4 1
Activated Partial Thromboplastin Time (APTT) 1.4 s 30 – 40 s
Fibrinogen 0.3 g/L 2 – 4 g/L
pH 7.24 7.35 – 7.45
P(CO2) 4.6 kPa 5.6 – 6.0 kPa
P(O2) 25.3 kPa 10 .5 – 13 .5 kPa
Base Excess (BE) -8.9 mmol/l -2 to +2.0 mmol/l
Bicarbonate 13 mmol/l 24 – 30 mmol/l

What is the most appropriate initial treatment?

a) 2 units of packed red cells.


b) 10 mg intravenous vitamin K.
c) 100 ml 8.4 % sodium bicarbonate.
d) 10 units cryoprecipitate.
e) 1 pool of platelets.

D:10 units cryoprecipitate.


The major abnormality is low fibrinogen level, which would also account for the abnormal INR and APTT. This is
likely to be due to a consumptive coagulopathy.

In the United Kingdom, fibrinogen is most commonly replaced with cryoprecipitate. Cryoprecipitate also contains
factor VIII, Factor XIII and von Willebrand factor.

A: 2 units packed red cells.


Although the dressing is bloodstained, the patient’s Hb level is adequate but needs to be monitored closely.

The British Society for Haematology guidelines for red cell use in critical care recommend that anaemic, critically
ill patients with stable angina should have their HB level maintained at > 70 g/L. In patients suffering from acute
coronary syndrome, the Hb level should be maintained at > 80 – 90 g/L.

B: 10 mg intravenous vitamin K.
In an acute setting, it would be more appropriate to correct the INR and APTT ratios by further FFP transfusion,
rather than by vitamin K administration, but the major abnormality is the low fibrinogen level correcting this may
correct these results also.

C: 100 ml 8.4 % sodium bicarbonate.


Although acidaemia may impair coagulation, this is not the primary abnormality in this case. The administration of
sodium bicarbonate has been associated with impaired haemostatic function in some studies.

E: 1 pool of platelets.
The platelet level is low, but not lower than 75 x 109/L, and the patient is not actively bleeding.

Thrombocytopenia is considered a late event in massive haemorrhage, typically seen only after a loss of at least 1.5
blood volumes.

44. A 55-year-old man requires a general anaesthetic for a procedure in the cardiac catheter laboratory. He has
a past medical history of type II diabetes, hypertension, hypercholesterolaemia and a previous transient
ischaemic attack. His medication includes atorvastatin, amlodipine, clopidogrel and metformin.

Which of the following best explains the mechanism of action of his antiplatelets agent?

a) Cyclo-oxygenase-1 enzyme inhibition.


b) Glycoprotein IIb/IIIa- receptor inhibition.
c) P2Y12 receptor inhibition.
d) Phosphodiesterase inhibitor.
e) Protease-activated receptor inhibition.

C: P2Y12 receptor inhibition.


Clopidogrel is a second generation thienopyridine adenosine diphosphate (ADP)-receptor-blocking drug. It
is a prodrug whose active metabolite acts on the P2Y 12 ADP receptor, inhibiting ADP-receptor-mediated
platelet aggregation. Its effects are permanent and irreversible for the duration of platelet lifespan (7- 10
days), which is important to remember when considering neuro-axial anaesthesia in a patient on an
antiplatelet agent.

A: Cyclo-oxygenase-1 enzyme inhibition.


Aspirin exerts it antiplatelets effects primarily via cyclo-oxygenase -1 (COD-1) inhibition. The enzyme COX-1 is
mainly responsible for thromboxane A2 synthesis, which stimulates platelet aggregation and localized
vasoconstriction.

B: glycoprotein IIb/IIIa- receptor inhibition


Tirofiban, eptifibatide and abcximab are all antiplatelet agents that act at the Glycoprotein IIB/IIIa receptor on
platelets. This is present on platelets in an inactive form. When the platelets are activated, they undergo a
conformational change, resulting in the binding of fibrinogen and von Willebrand factor (vWF). These molecules
function as a bridge for platelet aggregation. The GP IIb/IIIa-receptor blocking drugs bind to the GP IIB/IIIa
receptor and inhibit platelet aggregation by preventing the binding of vWF and fibrinogen.
D: Phosphodiesterase inhibition.
The antiplatelet drug dipyridamole exerts its antiplatelet effects by phosphodiesterase inhibition. This prevents the
degradation of cyclic adenosine monophosphate, which in turn, causes platelet activation.
E: Protease-activated receptor inhibition.
Thrombin activates platelets via the Protease-activated receptor (PAR). The novel antiplatelet agent vorapaxar acts
via the PAR-1 receptor.

45. A 50-year-old man presents to the Emergency Department as a trauma call following a road traffic
collision. The patient is hypotensive and shocked. He has a clear left leg deformity and head injury. The
patient is stabilized and taken for an emergency laparotomy. The ‘Massive-transfusion policy’ has been
activated, and the patient has received an 8-unit red-blood-cell transfusion. The Anaesthetist is concerned
the patient is developing a coagulopathy and uses Thromboelastography (TEG) to guide the administration
of further coagulation products. There is a concern for hyperfibrinolysis.

Which of the following values is most probably increased in this patient?

a) Alpha-angle.
b) Clot lysis at 30 minutes after maximum clot strength (LY30)
c) K time
d) Maximum amplitude (MA)
e) Reaction (R) time.

B: Clot lysis at 30 minutes after maximum clot strength (LY30)


The clot lysis at 30 minutes after maximum clot strength (LY30) is the percentage of amplitude reduction at 30
minutes after the maximum amplitude) MA – or clot strength) has been reached. An excessive reduction in the
LY30 (thus a greater percentage of amplitude reduction) suggests the presence of excess fibrinolysis, and
tranexamic acid may be indicated.

Thromboelastography (TEG) is a point-of-care test that gives an indication of clotting function. This test may be
able to elicit abnormalities and determine the cause of a coagulopathy. It is being increasingly used in theatres and
emergency departments. Traditional tests of clotting [prothrombin time (PT), and activated partial thromboplastin
time (APTT)] measure coagulation factor function, but TEG has the added advantage that it can also assess platelet
function, clot strength and fibrinolysis.

Understanding TEG graphs is important in determining what may be contributing to a coagulopathy.

https://youtu.be/JMmk8eSSp40?si=r4Ds_k14j61IIrEs
A: Alpha-angle.
The alpha-angle measures the speed of fibrin accumulation. A reduced alpha-angle may suggest a problem with
fibrinogen, and cryoprecipitate may be indicated.

C: K time.
The K time measures fibrinogen function. This is the time from the end of the R time until the clot reaches an
amplitude of 20 mm.
D: Maximum Amplitude (MA).
The MA is a marker of clot strength and thus a marker of platelet function. A reduced MA suggest the possible need
for a platelet transfusion.

E: Reaction (R) time


The R time is a marker of the coagulation cascade. The R-value represents the time until the first evidence of a clot
is detected, so an increased R time suggests a deficiency of coagulation factors.

46. A 48-year-old man is undergoing a radical nephrectomy for a renal cell tumour. On incision, he is noted to
be ‘oozy’. The surgeon confirms that prothrombin time (PT), activated partial thromboplastin time (APTT)
and platelets were normal on preoperative bloods: however, it was noted that his fibrinogen was only
0.8g/l.

Blood was taken to perform a thromboelastogram (TEG) and a separate test of platelet function. The
multiplate test confirmed that the platelet function was normal.

Normal ranges:

 Reaction time (R) = 20 – 30 mm


 K-time(K) = 8 – 15 mm
 Maximum amplitude (MA) = 50 – 60 mm
 Alpha-angle (A-angle) = 30 – 47 o

Which of the following options is most likely to represent this patient’s test result?

a) R = 23 mm; K = 5.5 mm; MA = 65 mm; A-angle = 50o


b) R = 21 mm; K = 25 mm’ MA = 28 mm; A-angle = 20 o
c) R = 17 mm; K = 6 mm; MA = 44 mm; A-angle = 47o
d) R = 14 mm; K = 55 mm; MA = 71 mm; A-angle = 50 o
e) R = 15 mm; K = 5 mm; MA = 67 mm: A-angle = 57o

B: R = 21 mm; K = 25 mm’ MA = 28 mm; A-angle = 20 o


This is correct and would be diagnostic of hypofibrinaemia.

The TEG is a near point-of-care test that gives important information with regard to coagulation of whole
blood.

There are five main parameters to consider in a TEG trace:

 R (Reaction time) = period of time from the start of the test to initial fibrin formation.
 K (clot kinetics) = measurement from the beginning of clot formation until the amplitude of the
TEG reaches 20 mm.
 A-angle = angle of a line drawn from the R-value + 1 mm and the K-value.
 MA = the greatest amplitude of the trace expressed as a percentage of the MA.

The following are the most likely diagnosis of the above options.
 1 = normal trace
 2 = hypofibrinogenemia
 3 = parameters with significant clot lysis likely to indicate poor platelet function and
fibrinolysis.
 4 = fictional
 5 = hypercoagulable state

Hence, this is correct. after a relatively normal R-time, hypofibrinogenemia causes a long thin, ‘champagne
glass’-shaped trace with a low A-angle and a low MA.

This could also represent a patient with thrombocytopenia or poor platelet function; however, in this case,
you have been told that the platelet function and number were normal.

C: R = 17 mm; K = 6 mm; MA = 44 mm; A-angle = 47o


This is diagnostic of poor platelet function – you know that platelet function is normal.

A: R = 23 mm; K = 5.5 mm; MA = 65 mm; A-angle = 50o


This set of results is essentially a near-normal trace.

D: R = 14 mm; K = 55 mm; MA = 71 mm; A-angle = 50 o


The K-time is marked deranged and is indicative of the time required to achieve a certain level of clot. It is
dependent on fibrinogen.

E: R = 15 mm; K = 5 mm; MA = 67 mm: A-angle = 57o


This is more likely to by hypercoagulable.

THROMBOELASTOGRAPHY

Thromboelastography (TEG) is a method of testing the efficiency of blood coagulation. It is a test mainly used
in surgery and anesthesiology, although increasingly used in resuscitations in emergency departments, intensive care
units, and labor and delivery suites. More common tests of blood coagulation include prothrombin time (PT)
and partial thromboplastin time (aPTT) which measure coagulation factor function, but TEG also can assess platelet
function, clot strength, and fibrinolysis which these other tests cannot.[1][2]

Thromboelastometry (TEM), previously named rotational thromboelastography (ROTEG) or rotational


thromboelastometry (ROTEM), is another version of TEG in which it is the sensor shaft, rather than the cup, that
rotates.

MECHANICS

A small sample of blood is taken from the selected person and rotated gently through 4º 45', six times a minute, to
imitate sluggish venous flow and activate coagulation. The clot forms around a thin wire probe used for
measurement. The speed and strength of clot formation is measured in various ways, typically by computer. The
speed at which the sample coagulates depends on the activity of the plasma coagulation
system, platelet function, fibrinolysis and other factors which can be affected by genetics, illness, environment, and
medications. The patterns of changes in strength and elasticity in the clot provide information about how well the
blood can perform hemostasis and how well or poorly different factors are contributing to clot formation.[2]
Four values that represent clot formation are determined by this test:

 the reaction time (R value)


 the K value,
 the angle and the
 maximum amplitude (MA).

The R value represents the time until the first evidence of a clot is detected.

The K value is the time from the end of R until the clot reaches 20mm and this represents the speed of clot
formation.

The angle is the tangent of the curve made as the K is reached and offers similar information to K.

The MA is a reflection of clot strength.

A mathematical formula determined by the manufacturer can be used to determine a Coagulation Index
(CI) (or overall assessment of coagulability) which takes into account the relative contribution of each of
these 4 values into 1 equation. The G-value is a log-derivation of the MA and is meant to also represent the
clot strength using dynes/sec as its units. There are some studies which suggest that an elevated G-value is
associated with a hypercoagulable state and therefore increases the risk for venous thromboembolic
disease. However, there are no studies dosing of prophylactic heparin products based on the G-value.

TEG also measures clot lysis which is reported as both the estimated percent lysis (EPL) and the percentage
of clot which has actually lysed after 30 minutes (LY 30,%). Although a normal EPL can be as high as 15%
and a normal LY 30 can be as high as 8%, some studies in the trauma population suggest that a LY30
greater than 3% is associated with risk of hemorrhage.[2][3]

PARAMETERS

Parameters derived from thromboelastography are mainly:[5]

 R time: Time to initial clot formation (that is, amplitude deviation from baseline)
 K time: Time from initial clot formation until reaching 20 mm in amplitude
 Alpha angle (α): Angle between the baseline at initial clot formation, and a tangent line that intersects
the tracing curve.
 Maximum amplitude (MA): Maximum deviation of tracing to baseline.
 A30: Amplitude 30 minutes after reaching maximum amplitude.
INTERPRETATION

ASSAYS TYPES
There are several types of assays that can be run using TEG: Standard (kaolin), RapidTEG, heparinase, Functional
Fibrinogen and PlateletMapping. A standard TEG is the most commonly ordered test and includes the parameters
noted above. A RapidTEG uses tissue factor in addition to kaolin thereby further speeding up the reaction. In this
assay, the R-value is replaced by the TEG-ACT value which is measured in seconds rather than in minutes. The
remainder of the TEG parameters do not differ between a standard and RapidTEG. A heparinase TEG is used to
assess for heparin-associated anticoagulation as the cause of hemorrhage. It is used most commonly
following cardiopulmonary bypass procedures where heparin is reversed using protamine intraoperatively. In
instances where a patient develops bleeding due to recurrent coagulopathy (usually shortly after arrival to the ICU),
the heparinase TEG can help quickly discern patients who can be treated with additional dosing of protamine versus
those who need to be taken back to the operating room for re-exploration. In this assay, a standard TEG is run twice
– once using the patient's blood only and another time using the patient's blood plus added heparinase. If the two
graphs are nearly the same, the cause of bleeding is not related to heparin rebound. However, if the R-time
associated with the heparinase-added specimen is significantly shorter than the R-time of the patient's blood without
added heparinase, the bleeding is likely due to heparin rebound and should respond to administration of protamine.
Lastly, the platelet map TEG aims to determine to what degree platelet function may be inhibited due to
pharmacologic inhibition of either the arachidonic acid (AA) or adenosine diphosphate (ADP) pathways. Aspirin
inhibits platelet function via the AA pathway while clopidogrel inhibits platelet function via the ADP pathway; thus,
this test can be used to determine the degree to which a patient is anticoagulated due to either medication. In this
assay, a standard TEG is run using patient's whole blood. Then, separate assays are run using the patient's blood with
added AA or ADP. The contribution of fibrin to the MA is subtracted using a mathematical formula. This allows
determination of the MA (AA) and MA (ADP), respectively. The difference between the patient's whole blood result
and AA/ADP added results are used to calculate the percent inhibition.

USE IN TREATMENT

Because the R value on the TEG represents the time it takes for clot formation to start, it is a reflection of
coagulation factor activity. Coagulation factors are essentially enzymes that drive clot formation. Thus, a
significantly prolonged R time could be treated with frozen plasma. The alpha angle represents the thrombin burst
and conversion of fibrinogen to fibrin. Thus, a depressed alpha angle could be treated with cryoprecipitate. 80% of
the MA is derived from platelet function whereas the remaining 20% is derived from fibrin. Thus, a significantly
depressed MA could be treated with platelet transfusion or medications that improve platelet function, such as
DDAVP. An elevated EPL or LY30 suggests fibrinolysis and may be treated with an antifibrinolytic, such as
tranexamic acid or aminocaproic acid, in the appropriate clinical setting. A single, modified TEG assay with
exogenous tissue plasminogen activator (tPA) demonstrated remarkable efficiency in unmasking patients' impending
risk for massive transfusion in trauma patients.[8]

Clinical studies of thromboelastography during elective surgery (cardiac and liver surgery) and emergency
resuscitation have shown improvements in clinical outcomes. [9] In elective surgery there was a decreased need for
blood products (platelets and plasma) and reduced operating room length of stay as well as duration of intensive care
admission and bleeding rates; mortality was not affected. In emergency settings, mortality was reduced with an
associated decrease in the need for platelets and plasma.[9]

Additional studies show thromboelastography may be used to characterize COVID-19-associated coagulopathy.


TEG with platelet mapping may be used to guide use of anticoagulant and antiplatelet medications. When using a
TEG-guided strategy hospital length of stay, intensive care unit length of stay, mortality, acute kidney injury,
intensive care unit admissions and need for mechanical ventilation may be reduced. [10]

PERIOPERATIVE ANTICOAGULATION MANAGEMENT


Management of perioperative anticoagulation requires an interprofessional approach to determine the duration and
discontinuation of temporary interruption of anticoagulation. Parenteral anticoagulation during temporary
interruption may be required in certain circumstances based on the patient's and procedure's individualized risks and
benefits. This activity explains the recommendations for perioperative management of anticoagulation for patients
with nonvalvular atrial fibrillation undergoing non-cardiac surgery.
Objectives:

 Review the basic pharmacology that can affect the homeostasis in patients undergoing non-cardiac elective
surgery.
 Review the current indications for antithrombotic and anticoagulation interruption in patients undergoing
non-cardiac elective surgery.
 Describe the different emergency anticoagulation reversal treatments and their indications.
 Outline the importance of collaboration and coordination among the interprofessional team that can
enhance patient care when dosing and monitoring anticoagulant drugs to improve patient outcomes for
patients receiving anticoagulation undergoing non-cardiac elective surgery.

INTRODUCTION
The management of patients on anticoagulation and anti-aggregation therapy is a daily challenge for physicians. The
interruption of therapy can increase the risk of thrombotic events during and after surgery. However, the non-
interruption of these medications can heighten the risk of bleeding during surgery and trigger a sequence of
undesirable outcomes ranging from minor to uncontrolled bleeding.
The optimal management of these patients is thus achieved through a balance between thromboembolic and bleeding
risks. Several case-based considerations affect the decision of whether or not to interrupt anticoagulation or anti-
aggregation therapy before surgery. These include an evaluation of an individual's underlying bleeding risk, the risk
of bleeding associated with the surgical procedure, the timing of interruption and resumption of anticoagulation
therapy, and whether to use bridging therapy. These are all typical questions that will be addressed in this review.

AETIOLOGY

Anticoagulation therapy is most commonly indicated in the presence of atrial fibrillation (AF), deep venous
thrombosis (DVT), pulmonary embolism (PE), and after placement of prosthetic heart valves. Patients who have
undergone percutaneous coronary interventions are typically on dual antithrombotic therapy, as well as patients with
a past medical history of stroke, coronary artery by-pass grafting and essential thrombocytosis could require
antithrombotic therapy.

EPIDEMIOLOGY

AF, DVT, and PE are the leading causes of anticoagulation treatment. In the United States (U.S.) alone,
approximately 3 to 5 million people suffer from atrial fibrillation, and this number is expected to increase to 8
million by 2050.[1] Likewise, approximately 250,000 patients annually in the U.S. require anticoagulation therapy
cessation in order to be considered for surgery.[2]

PATHOPHYSIOLOGY

The process of physiologic hemostasis can be altered for several reasons, such as genetic disorders, malignancy,
sepsis, surgery, and drugs (those used for anti-aggregation and anticoagulation). From a basic approach to
perioperative anticoagulation, a pharmacological review of anticoagulation is essential.
Aspirin (Acetylsalicylic Acid)
This agent is the most commonly prescribed antiplatelet drug for the prevention of cardiovascular disorders. Its
mechanism of action is the irreversible inhibition of the cyclooxygenase (COX) 1 and 2 enzymes. The action of
COX is necessary for the conversion of arachidonic acid to prostaglandin (PG) H2. The PGH2 is rapidly converted
to several bioactive prostanoids, including thromboxane A2, a potent vasoconstrictor, and an inductor of platelet
aggregation. Despite the short half-life of aspirin (3 to 6 hours), its irreversible effects will last for the complete
lifetime of the platelet (8 to 9 days). After the interruption of aspirin therapy, recovery of platelet function depends
on its turnover (approximately 10% per day).[3][4]
Non-steroidal Anti-inflammatory Drugs (NSAIDs)
These drugs act by the inhibition of COX enzymes, depending on the particular drug. Some NSAIDs can selectively
inhibit the COX 2 enzyme that mediates pain and inflammation, simultaneously limiting the undesirable effects of
COX 1 inhibition. The effect of NSAIDs on platelet function is considered a short-term effect that normalizes within
three days; nonetheless, this can vary between drugs in the class. For short-acting drugs like ibuprofen, diclofenac,
and indomethacin, 50% of platelet function is restored 6 hours after the last dose and normalized after 24 hours.[4]
[5]
Thienopyridines (Clopidogrel and Prasugrel)
These are inhibitors of the adenosine diphosphate (ADP) receptor, also called the P2Y12 receptor, in platelets.
Physiologically, the union of ADP with its receptor increases levels of intracellular calcium and activates the
GpIIB/IIIa platelet receptor that promotes the stabilization of the platelet clot through fibrinogen bonds.
[6] Clopidogrel and prasugrel are prodrugs in which active metabolites irreversibly affect the platelet function in a
time- and dose-dependent fashion. A clopidogrel dose of 75 mg daily produces a 60% decrease of platelet function
in 3 to 5 days; in contrast, a 600 mg loading dose can achieve a steady-state platelet inhibition in 6 to 8 hours.[3] In
the same way, a loading dose of prasugrel of 60 mg is enough to produce steady-state platelet inhibition two hours
after drug administration. Due to the irreversible mechanism of action, it is recommended to interrupt these drugs 5
to 7 days before non-cardiac elective surgery.[4]

Non-thienopyridines (Ticagrelor and Cangrelor)


These drugs are relatively new with different mechanisms of action from those discussed thus far. Ticagrelor is a
reversible, non-competitive ATP analog that binds to a G-protein in the P2Y12 receptor, preventing its activation
and signaling. After a loading dose of ticagrelor, the maximum antiplatelet effect is achieved within 2 hours, plasma
half-life is 8 to 12 hours, and steady-state concentration in 2 to 3 days. Due to the reversible effect of ticagrelor on
platelets, it is recommended to be suspended 5 days before surgery.[4] On the other hand, cangrelor is a direct,
reversible, and intravenously administered drug that inhibits the P2Y12 receptor. Cangrelor can inhibit 95% to 100%
of platelet activity within the first two minutes of administration; the plasma half-life of cangrelor (3 to 6 minutes)
allows recovery of 80% to 90% platelet function within 60 to 90 minutes of discontinuing the intravenous infusion.
[7]
Vitamin K Antagonists (Warfarin, Acenocoumarol, Phenprocoumon)
These are also called coumarins. The most recognized and widely used drug of this group is warfarin, which has
been available for more than 50 years. The mechanism of action of warfarin is the inhibition of the 2,3 epoxide
reductase enzyme, responsible for the cyclical conversion of oxidized vitamin K to a reduced state. The latter is
necessary as a cofactor for the carboxylation of glutamic acid at the N-terminus of coagulation factors. Without
gamma carboxyglutamate residues, clotting factors II, VII, IX, and X cannot bind to the divalent calcium necessary
for normal activation. However, the inhibition of carboxylation also affects the production of protein C and S
anticoagulants. This creates a transient procoagulant state that can be explained by the shorter half-life of these
anticoagulants (8 and 30 hours), compared to factor II and factor X (60 and 72 hours). This phenomenon is more
frequent, with higher doses of vitamin K antagonists at the beginning of anticoagulation therapy. [8] Warfarin is a
racemic mixture of the R and S stereoisomers of the drug; the S isomer is 3-5 times more potent than the R isomer.
The half-life of warfarin is 36 to 42 hours (S isomer 29 hours, R isomer 45 hours); nonetheless, it can be altered by
several factors. In practice, warfarin is a drug of difficult titration due to the high number of pharmacological
interactions and genetic variations that can affect its metabolism.[9]
Direct Inhibitors of Factor Xa (Rivaroxaban, Apixaban, Edoxaban, Betrixaban)
The common mechanism of action these drugs, also known as direct oral anticoagulants (DOACs), is to bind to the
active site of factor Xa, thus inactivating it. Factor Xa is considered the rate-limiting step for the progression of the
coagulation cascade, thrombin activation, and ultimately clot formation.[10] Some advantages of prescribing
DOACs are their short half-life and rapid onset of action, allowing for an easier interruption and re-initiation of
anticoagulation therapy after surgery. Furthermore, it seems that the direct inhibitors of factor Xa have a lower
bleeding risk compared to vitamin K antagonists, making the use of routine coagulation tests unnecessary.
[11] However, the pharmacokinetic properties of each DOAC can vary according to the renal and liver function of
the patient (see table 1).
Direct Inhibitors of Thrombin
Dabigatran is the only medication in this group. Its mechanism of action is the direct inhibition of thrombin,
preventing the conversion of fibrinogen to fibrin and thus clot formation. Dabigatran has a quick onset of action (0.5
to 2 hours) and is metabolized by non-specific plasma esterases. The plasma half-life is around 12 hours; however,
the half-life of this drug is tremendously affected by renal function, as its excretion is 80% by the kidneys and less
than 10% by the liver. It is recommended to avoid dabigatran use with creatinine clearance (CrCl) less than
30ml/min, due to the potential for drug accumulation and adverse effect of bleeding (see table.1).
Fondaparinux
This drug is a pentasaccharide that acts as an indirect factor Xa inhibitor. The exact mechanism of action is the
reversible binding of the drug to the antithrombin factor, potentiating its activity to inactivate Factor Xa. The plasma
half-life of fondaparinux is approximately 15 to 17 hours. Its anticoagulant activity persists even 2 to 4 days after the
last dose of the drug in a person with normal renal function.[11]
Heparins
The binding of the heparin molecule to the antithrombin receptor enhances its potency to inactivate factors II and
Xa. This drug has been widely used for years, with multiple indications and dosages. One of the most concerning
adverse effects of this pharmacological group is the possibility of heparin-induced thrombocytopenia. Fortunately,
this complication is infrequent, though it is dependent on the dose, route of administration, and type of heparin. In
patients with renal failure and a CrCl < 30 ml/min, low molecular weight heparin (LMWH) should be avoided or
adjusted to renal function, the use of unfractionated heparin (UFH) is a suitable option in this case. Heparin can be
administered as a therapeutic dose for the patient with high thromboembolic risk (enoxaparin 1 mg/kg twice daily or
dalteparin 100 units/kg twice daily) or as the prophylactic dose for patients undergoing bridge therapy (enoxaparin
40 mg once daily or dalteparin 5,000 units once daily).[12]
Go to:

Evaluation

The approach recommended by several well-known guidelines is based on four considerations that aim to guide the
physician in elective surgery cases.[13]
Determine the Thromboembolic Risk of the Patient
The three major conditions related to a higher risk of thromboembolic events are atrial fibrillation, prosthetic heart
valves, and recent thromboembolism (venous or arterial). The thromboembolic risk will be estimated through
clinical variables in the case of atrial fibrillation with the CHA2DS2VASc score.[14] The localization, type of valve,
number of prosthetic valves, and associated cardiac risk factors will lead to the risk stratification of patients with
prosthetic valves. Regarding thromboembolism, the time after the episode and risk of recurrence will determine the
categorization of risk. The venous thromboembolism (VTE) can be classified as provoked (higher risk of
recurrence) when an inciting event is identifiable, or unprovoked when a cause is non-identifiable. An example of
provoked VTE is the patient with permanent risk factors such as congestive heart failure, malignancy inherited
thrombophilia, or inflammatory bowel disease.[15](see Table.2)
Determine the Procedural Bleeding Risk
When evaluating bleeding risk, it is imperative to consider the type of surgery and clinical characteristics of the
patient. These characteristics are assessed with the HAS-BLED (Hypertension, Abnormal liver or kidney function,
Stroke, Bleeding history or predisposition, Labile International Normalized Ratio [INR], elderly, drugs, alcohol)
score. Each positive item earns 1 point, and a HAS-BLED score > 3 indicates a high bleeding risk. The BNK Online
Bridging Registry (BORDER) study assessed the possible predictors of perioperative bleeding in patients
undergoing invasive cardiac procedures. They found that the HAS-BLED score is a reliable predictor of
perioperative bleeding (HR 11.8, 95% CI 5.6-24.9).[16] As a general guideline, the procedural risk can be divided
into low risk (0-2% two-day risk of major bleeding) or high risk (2% to 4% two-day risk of major bleeding). Among
the diversity of surgical procedures, intracranial, cardiac, and neuraxial procedures are of special concern due to the
localization of potential bleeding and poor patient outcomes in the presence of this complication.[17] (see table 3)
Determine Whether or Not Interrupt Anticoagulation or Antithrombotic TherapyThe balance between risks
and benefits of anticoagulation interruption is the right approach to this decision. Clinical judgment is imperative, as
there is no score or calculator to determine the classification of the patient straightforwardly.
As a generalized consideration, patients with a high risk of bleeding will benefit from anticoagulation interruption.
[18][19][20] However, those patients with high thromboembolic risk might benefit from bridging therapy and the
shortest period of anticoagulation withdrawal as possible. An example of this scenario is the patient undergoing
potentially curative cancer surgery. In other scenarios, the delay of the elective surgical procedure after a balance of
risks and benefits is a suitable option.
In patients with a recent episode of VTE (less than 1 month ago), the risk of recurrence in the subsequent month can
be as high as 40%. An elective surgical procedure should be deferred up to 3 months after an episode of VTE, if
possible. For patients with a recent acute ischemic stroke undergoing non-cardiac elective surgical procedures, the
risk of having a major cardiovascular event after surgery is high, especially within the first 3 months. The American
College of Surgeons recommends deferring the surgery up to 9 months in this scenario when the risk of cardiac
events has plateaued after a stroke.[18]
Another common scenario is the patient with coronary stents, most of which are on dual anti-aggregation.
Approximately 5% of these patients will require surgery during the next year after coronary stent implantation. If
aspirin and thienopyridine need to be suspended during the perioperative period, the elective surgical procedure
should be delayed 6 weeks for patients with bare-metal stents and 6 months for drug-eluting stents.[4][12] In case
the elective surgery cannot be postponed, the dual anti-aggregation therapy should be continued throughout the
perioperative period.[21][22]
Despite limited evidence, there are scenarios where anticoagulation therapy continuation is preferable in patients
with low bleeding risk. Dental procedures are considered low bleeding risk procedures. The majority of evidence
comes from patients receiving warfarin with a therapeutic INR range. The studies conclude that continuing
anticoagulation therapy is safe, not only in dental extraction but in other types of dental procedures and that bridging
therapy could be related to even more bleeding associated with these procedures.[23][24] One four-year, cross-
sectional study assessed bleeding risk in patients with DOACs, showing no significant bleeding association with
continuing anticoagulation therapy.
Endovascular procedures (venous and arterial angioplasty, angiography, catheter ablation for AF, and atherectomy)
studies were analyzed in a meta-analysis of 20,000 patients on warfarin anticoagulation. [25][26] The study
concluded that rates of complications were low or equivalent between interrupted and uninterrupted warfarin
anticoagulation groups. Similar findings have been published with DOACs. The first study was the VENTURE-AF
(rivaroxaban vs. VKA in patients with AF undergoing catheter ablation); the number of complications was low and
similar between groups.[27] The RE-CIRCUIT study (dabigatran vs. warfarin in patients with AF undergoing
catheter ablation); dabigatran had lower rates of major bleeding than warfarin, and there was no difference in stroke
or systemic embolism between groups.[28] Uninterrupted apixaban and edoxaban have also been shown to be safe
in patients with AF undergoing catheter ablation.
Implantation of prosthetic cardiac devices is also considered low bleeding risk procedures. Two studies, the
BRUISE CONTROL 1 and 2, have addressed this scenario. The first study examined uninterrupted anticoagulation
vs. warfarin interruption and a low therapeutic dose of low molecular weight heparin (LMWH) (1 mg per kg/twice
daily of enoxaparin) in patients undergoing pacemaker implantation. The patients with uninterrupted warfarin had a
lower incidence of pocket hematoma complications compared with the group that received bridging therapy with
LMWH (3.5% vs. 16%, P <0.01).[29] The BRUISE CONTROL 2 study evaluated the safety of performing
electrophysiologic device procedures without interrupting DOAC anticoagulation therapy. The study compared
uninterrupted DOACs vs. interrupted DOACs (apixaban, rivaroxaban, dabigatran interrupted two days prior to
surgery) in patients with high thromboembolic risk. The strategy of continuing DOACs for reducing the
complication of pocket hematoma was not superior to the interruption of DOACs 48 hours prior to the procedure.
[30] Additionally, there was no statistically significant difference in other outcomes such as stroke, any hematoma,
and any adverse event. This trial was terminated early due to futility.
Determine Whether or Not Bridging Anticoagulation
Bridging anticoagulation consists of the substitution of a long-acting anticoagulant (usually with warfarin) for a
shorter-acting anticoagulant (usually LMWH) to limit the time of subtherapeutic anticoagulation levels and
minimize thromboembolic risk[31]. Despite the growing evidence about the limited to nonexistent benefits of
bridging therapy, it is still being used on a case-by-case basis. Clinical scenarios that may benefit from bridging
therapy are those involving patients with high thromboembolic risk. In several guidelines, the following scenarios
have been proposed:

 The patient with a mechanical heart valve: Mitral valve replacement, aortic valve replacement with
additional risk factors (stroke, TIA, cardioembolic event, or intracardiac thrombus), more than 2
mechanical valves.
 Patients with stroke, episode of systemic emboli, or VTE during the last 3 months. Patients presenting with
a thromboembolic event after interruption of chronic anticoagulation therapy or those presenting with VTE
while on therapeutic anticoagulation.
 Patients with atrial fibrillation and CHA2DS2VASc score > 5 plus additional cardiovascular risk factors
(rheumatic valve disease, stroke, or systemic embolism within the last 12 weeks). A CHA2DS2VASc score
> 6 with or without additional risk factors.
 Patients with recent coronary stenting (within the previous 12 weeks)
Evidence on bridging therapy follows a trend against its use. Sigeal et al. (2012) performed a meta-analysis of 34
studies, most of them were observational. Despite the criticism of this meta-analysis for its heterogeneity of studies,
interesting conclusions were drawn. Regarding the risk of thromboembolism, there was no statistically significant
difference between patients that received bridging therapy and those who did not receive bridging therapy(OR:0.80;
95% CI 0.42-1.54). However, the risk of major bleeding had a threefold increase in patients that received bridging
therapy compared with patients who did not (OR: 3.60; 95% CI 1.52-8.50).[32]
Large studies of anticoagulation in patients with atrial fibrillation, such as the RE-LY study (warfarin vs.
dabigatran), showed that patients on warfarin who received bridging therapy had more thromboembolic events
compared with those that did not receive bridging therapy (1.8 vs. 0.3%). Patients that received warfarin plus
bridging therapy had a higher risk of major bleeding (warfarin plus bridging therapy 6.8%; warfarin without
bridging 1.6%), compared with patients that received dabigatran plus bridging therapy (dabigatran plus bridging
therapy 6.5%; dabigatran without bridging therapy 1.8%).[33]
In the BRIDGE (Bridging anticoagulation in patients who require temporary interruption of warfarin therapy for
elective surgery) study, 1,884 patients with AF requiring interruption of warfarin anticoagulation therapy were
allocated to either a dalteparin or placebo group. There was no difference between groups in arterial
thromboembolic events at 30 days after surgery (0.3 vs. 0.4). However, the incidence of major bleeding was higher
in the dalteparin bridging group (3.2% vs. 1.3%, P <0.01), as was minor bleeding (20.9% vs. 12.0%, P <0.01). This
study was criticized for the exclusion of high thromboembolic risk patients.[34]
The PERIOP-2 study examined whether there is any benefit of bridging therapy after surgery. This study was a
randomized clinical trial of 1,471 enrolled patients, 304 of which had a mechanical valve (99 of these had AF at the
same time). There were no significant differences for major thromboembolic episodes between dalteparin and
placebo (0.71% vs. 1.11%, respectively). For episodes of major bleeding, this study showed no benefit of using
dalteparin vs. placebo during the postoperative period (1.46% vs. 2.46%, respectively).

TREATMENT / MANAGEMENT
How to bridge
Our bridging recommendations follow the AT9 guidelines, which agree with the American Society of Regional
Anesthesia (ASRA) 2018 guidelines and are similar to the American College of Surgeons 2018 guidelines. [4][12]
[18]
During the preoperative period:

 Discontinue warfarin five days before surgery.


 Three days before surgery, start subcutaneous LMWH or unfractionated heparin (UFH), depending on the
renal function of the patient at therapeutic doses.
 Two days before surgery assess INR, if greater than 1.5 vitamin K can be administered at a dose of 1 to 2
mg.
 Discontinue LMWH 24 hours before surgery or 4 to 6 hours before surgery if UFH.
During the postoperative period:

 If the patient is tolerating oral intake, and there are no unexpected surgical issues that would increase
bleeding risk, restart warfarin 12 to 24 hours after surgery.
 If the patient received preoperative bridging therapy (high thromboembolic risk) and underwent a minor
surgical procedure, resume LMWH or UFH 24 hours after surgery. If the patient underwent a major
surgical procedure, resume LMWH or UFH 48 to 72 hours after surgery.
 Always assess the bleeding risk and adequacy of homeostasis before the resumption of LMWH or UFH[35]
How to Manage Patients on DOAC Anticoagulation Therapy Undergoing Elective S urgery
It is important to note that bridging therapy is not indicated in patients on DOACs. The predictable pharmacological
effect of DOACs allows a properly timed interruption of anticoagulation therapy before surgery. Various societies
have issued recommendations about the timing of DOAC interruption. Our recommendations are based on the
American College of Cardiology Expert Consensus (2017), European Heart and Rhythm Association (2018),
American Society of Regional Anesthesia (ASRA) (2018).[4][19][20]
The appropriate timing interruption for patients on DOAC anticoagulation is based on the invasiveness and bleeding
risk of the procedure, pharmacokinetic profile of the DOAC, and clinical characteristics of the patient (renal
function and liver function, see Table.4). As a common recommendation among guidelines, DOACs should be held
3 half-life times before low-risk procedures and 5 half-life times before high-risk procedures. Nevertheless, there are
some procedures considered less than low bleeding risk, such as a colonoscopy without biopsy, where DOAC
therapy may be continued.(see table.4)
Betrixaban is a new inhibitor of factor Xa. Though many aspects of the clinical use of this drug are still under study,
ASRA (2018) recommends betrixaban interruption at least 72 hours before a neuraxial block. ASRA also
recommends at least 5 hours between catheter removal and reinitiation of the drug.[36]
In 2019, a new strategy was published in the PAUSE study, a prospective clinical trial evaluating a standardized
approach for perioperative management of DOACs. The interruption scheme used in this study was simple. For high
bleeding risk procedures, rivaroxaban, apixaban, and dabigatran were suspended 48 hours before surgery in patients
with CrCl>50 ml/min. If the renal function was compromised (CrCl< 50 ml/min), these drugs were interrupted for
four days before surgery. For low bleeding risk procedures, rivaroxaban, apixaban, and dabigatran were interrupted
24 hours before surgery in patients with CrCl>50 ml/min. If the renal function was compromised (CrCl <50
ml/min), drugs were suspended two days before the procedure. Regardless of renal function, all drugs were
reinitiated at 48 hours for high bleeding risk surgical procedures and 24 hours for low bleeding risk procedures. The
30-day postoperative rate of major bleeding was 1.35% (95% CI, 0%-2.00%) and rate of arterial thromboembolism
of 0.16% (95% CI, 0%-0.48%). However, more studies are needed in patients with high surgical bleeding risk.[37]
How to Manage Antithrombotic Therapy in Patients Undergoing Elective Surgery
Ideally, patients with recent coronary stent implantation should postpone elective surgery during critical periods
post-placement (6 weeks for bare metal stents and 6 months for drug-eluting stents). If elective surgery cannot be
postponed, dual antithrombotic therapy should be continued throughout the perioperative period. In patients at high
risk of cardiac events, aspirin should be continued throughout the perioperative period, with clopidogrel and
prasugrel discontinued 5 days prior to surgery and resumed 24 hours postoperatively. For patients at low risk of
cardiac events, dual antiplatelet therapy can be discontinued 7 to 10 days before surgery and restarted 24 hours
postoperatively.[4]
Considerations for Patients Undergoing Neuraxial Anesthesia
The following recommendations for neuraxial puncture and catheter removal are based on the European Society of
Anesthesiology and ASRA. Included in these recommendations are the timelines for interrupting and resuming
anticoagulation after the puncture, catheter manipulation, or removal.
For patients treated with intravenous UFH, the infusion should be interrupted 4-6 hours before puncture or catheter
removal/manipulation. However, if the patient is receiving UFH subcutaneously, the drug should be interrupted 8-12
hours before puncture or catheter removal/manipulation. Independent of the route of UFH administration, the drug
can be restarted 1 hour after puncture or catheter removal/manipulation.
For patients receiving a prophylactic dose of LMWH, the drug should be interrupted for 12 hours before puncture or
catheter removal/manipulation and 24 hours if the dose is therapeutic. Independent of LMWH dose, the drug may be
restarted 4 hours after puncture or catheter removal/manipulation. If the patient is receiving fondaparinux at a
prophylactic dose (2.5 mg/day), it should be stopped 36-42 hours before any neuraxial approach and might be
resumed 6 to 12 hours post-procedure.
For patients receiving DOACs, the time of interruption will vary according to the specific drug. Patients on
rivaroxaban at a prophylactic dose (less than 10 mg/day) should have the drug interrupted 22 to 26 hours before a
neuraxial approach. If the patient is on apixaban at a prophylactic dose, it should be discontinued 26 to 30 hours
before a neuraxial puncture or catheter placement. Both of these drugs may be restarted 4 to 6 hours after the
puncture or catheter removal/manipulation. If the patient is on coumarin anticoagulation, the INR should be less
than 1.4 before any neuraxial approach.
Drugs such as NSAIDs, including aspirin, do not require interruption for neuraxial anesthetic interventions.
However, others such as cilostazol will require 42 hours of an interruption prior to neuraxial anesthesia and can be
resumed 5 hours after the puncture or catheter removal.[4]
How to Perform Emergency Reversal of Anticoagulation
Definitions of emergency and urgent surgery can vary slightly between guidelines. An urgent procedure is defined
as one that can be delayed up to 24 hours, giving time to the physician to conduct anticoagulation reversal based on
repeated coagulation tests. On the other hand, the emergent scenario can be defined as one in which the patient
requires surgical intervention in less than 1 hour or is experiencing life-threatening bleeding. In emergent cases, the
amount of time, number of coagulation tests that can be performed, and opportunities to delay the procedure until
the bleeding risk is lower are limited.[38]
Reversal of warfarin: For non-significant bleeding with alterations of the INR, conservative strategies such as
interruption of the drug and oral vitamin K are suitable options. In the emergent scenario, the reversal of warfarin
anticoagulation is based on prothrombin complex concentrate (PCC) and fresh frozen plasma (FFP) administration
as follows:

 INR 2-4: PCC 25 IU/kg IV


 INR ≥4-6: PCC 35 IU/kg IV
 INR >6: PCC 50 IU/kg IV

 Vitamin K: 10 mg IV administered slowly


 FFP: 10 to 20 ml/kg
 Trauma patients: 1 gm of tranexamic acid can be used at arrival and repeat dose of 1 gm in 8 hours
 PCC is commercially available as prothrombin complex concentrate both contain heparin and are thus
contraindicated in patients with a past medical history of heparin-induced thrombocytopenia.[13]
The required time for anticoagulant reversal and INR correction with vitamin K and PCC are 6 to 24 hours and 15
minutes after 10-minute to 1-hour infusion, respectively. FFP should be administered if there is ongoing bleeding or
PCC is not available. The timing of anticoagulation reversal depends on the timing to complete infusion. Complete
reversal usually takes up to 32 hours. PCC must be administered with vitamin k. [39]
Reversal of DOACs: The FDA recently approved andexanet alfa for the reversal of the anticoagulative effects of
apixaban and rivaroxaban.[40] The effects of andexanet alfa also seem to be extended to betrixaban and edoxaban;
however, more studies are needed. DOAC anticoagulation represents a problem for the clinician. For several years,
the first-line option for the reversal of anticoagulation was PCC. However, evidence about prothrombin complex
concentrate effectiveness is controversial, and with the approval of new specific reversal medications, its future use
is uncertain (see table 5). What is known is that the cost of the specific new reversal drugs is still high, limiting their
use and widespread availability. A complete reversal treatment with andexanet alfa can cost around $50,000,
compared to the cost of full treatment with prothrombin complex concentrate, about $5,000.[41]

DIFFERENTIAL DIAGNOSIS

 Acute Anemia

 Afibrinogenemia

 Child abuse

 Dysfibrinogenemia

 Epistaxis

 Factor V deficiency

 Factor X deficiency

 GI bleeding

 Idiopathic Thrombocytopenic Purpura

 Liver failure

 Munchausen syndrome

 Subdural hematoma

 Type A haemophilia

 Type B haemophilia

COMPLICATIONS
There are two major complications of poor management of perioperative anticoagulation. The first is bleeding,
which occurs if the provider fails to interrupt anticoagulation therapy in an appropriate timeframe. On the other
hand, however, patients who have their anticoagulation interrupted too early in the perioperative period are at high
risk of thromboembolic events, as surgical procedures themselves induce a hypercoagulable state. Thus, appropriate
interruption of anticoagulation in the perioperative period is a delicate balancing act between the potentially severe
complications of bleeding and thrombosis, requiring strict attentiveness of the managing provider.

ENHANCING HEALTHCARE TEAM OUTCOMES


Managing perioperatively the patient receiving anticoagulation therapy is the responsibility across all disciplines of
the healthcare team. First, it is the responsibility of the nurse to ensure the information on anticoagulation therapy
and diagnoses is correct for the patient upon admission for preoperative testing/physical exam. During the
preoperative visit, the surgeon and anesthesiologist in charge then have the responsibility of ensuring this
information is correct and then deciding how the patient will be managed perioperatively. Once that decision is
made, it is the responsibility of the physician to explain this information in detail, the nurse to review this before
discharge, and finally, the pharmacist to answer any questions and/or reiterate this information.
On the day of surgery, operative nurses, physicians, and pharmacy personnel have their usual duties of ensuring the
patient receives appropriate operative care per protocol for the procedure. Postoperatively, the physician has the
responsibility of placing orders for either continuing to hold to administering the anticoagulation agent, depending
on the drug and the patient scenario. However, it is the duty of the recovery room and hospitalization floor nurses to
re-check each prescription before administration according to the diagnoses of the patient. Additionally, pharmacy
personnel will be in charge of review the pharmacological conciliation of the patient.
Finally, it will be time for the patient to be discharged. The physician will be responsible for writing discharge
orders, which may consist of continuing the same anticoagulation regimen that was prescribed preoperatively,
changes to the dosage, or even holding medication for some time. Such orders will again be checked by pharmacy
personnel. In general, it is the responsibility of the nurse to ensure that discharge instructions are received and
understood by the patient, thus completing the interdisciplinary care circle for the patient receiving anticoagulation
perioperatively. Only by working as an interprofessional team can the morbidity and mortality related to
anticoagulation treatments and physicians errors be diminished. [Level 5]

You might also like